Kidney Flashcards

1
Q
  1. Oncocytosis is considered in a 50-year-old man with bilateral renal masses. Which is the single best answer regarding oncocytosis?

A. Account for 2% of all renal cortical tumours

B. Multifocality bilateralism and metachronous tumours occur together in 4-6%

C. In multifocal cases co-existent RCC is present in 50% of cases

D. Often have preserved renal function

E. The final diagnosis can be made on MR above

A

B. Multifocality bilateralism and metachronous tumours occur together in 4-6%

Co-existent RCC in 10% of cases. Due to diffuse bilateralrenal involvement patients with oncocytosis often presentwith abnormal renal function. Final diagnosis is by biopsy.

How well did you know this?
1
Not at all
2
3
4
5
Perfectly
2
Q
  1. In a 40-year-old woman with seizures and bilateral renal masses, with the appearance of multiple angiomyolipomas, which is the single best answer?

A. Sporadic solitary Angiomyolipoma (AML) have a female preponderance

B. Account for 50% of all renal AMLs

C. Are associated with tuberous sclerosis

D. Warrant treatment due to risk of bleeding when > 2cm

E. Are typically hypoechoic on US

A

C. Are associated with tuberous sclerosis

Those AML associated with tuberous sclerosis are multifocal, bilateral, larger and present in a younger age group. AML appear echogenic on US.

How well did you know this?
1
Not at all
2
3
4
5
Perfectly
3
Q
  1. Which is a cause of dilated calyx with wide infundibuli?

A. Hydrocalycosis

B. Structure secondary to calculus

C. Structure secondary to TB

D. Extrinsic compression by an artery

E. Post-obstructive uropathy

A

A. Hydrocalycosis

Answers B-E are causes with narrow infundibuli

How well did you know this?
1
Not at all
2
3
4
5
Perfectly
4
Q
  1. During MR of the spine a 2cm renal lesion is identifed at the upper pole cortex of the RK. The lesion is well-defned and is of high SI on T1 and T2 images. Ultrasound shows normal Doppler flow in the renal vein which is separate from the lesion, which is echogenic on US. The fndings are most likely to represent which of the following?

A. AML

B. Atypical cyst

C. Abscess

D. RCC

E. Lymphoma

A

A. AML

AML appears increased on S1 on T1+T2 and is echogenic onUS.

How well did you know this?
1
Not at all
2
3
4
5
Perfectly
5
Q
  1. Which of the following causes immediate faint persistent nephrogram on Intravenous Urogram (IVU)?

A. Renal vein thrombosis (RVT)

B. Acute obstruction

C. Acute hypotension

D. Acute tubular necrosis

E. Chronic obstruction

A

A. Renal vein thrombosis (RVT)

Acute glomerulonephritis, RVT and chronic severe ischaemia are all causes of immediate faint persistent nephrogram.

How well did you know this?
1
Not at all
2
3
4
5
Perfectly
6
Q
  1. Which is the cause of rim nephrogram?

A. Acute complete arterial occlusion

B. Acute ureteric obstruction

C. Polycystic kidney disease

D. Medullary sponge kidney

E. Acute pyelonephritis

A

A. Acute complete arterial occlusion

Acute complete arterial occlusion and severe hydronephrosis are causes of a rim nephrogram. B-E are causes of a striated nephrogram.

How well did you know this?
1
Not at all
2
3
4
5
Perfectly
7
Q
  1. A 45-year-old female with a six-month history of urinary tract symptoms undergoes CT showing an enlarged right kidney with a large staghorn calculus and hydronephrosis. The renal parenchyma is replaced by multiple confluent fluid-flled masses. Open nephrectomy histology from the right kidney reveals lipid laden foamy macrophages in combination with an inflammatory granuloma and a lymphoplasmocytic infltrate. What is the most likely diagnosis?

A. Renal lymphoma

B. Xanthogranulomatous pyelonephritis

C. Renal TB

D. Malakoplakia

E. Nephrocalcinosis

A

B. Xanthogranulomatous pyelonephritis

A rare form of low-grade chronic renal infection with progressive destruction of renal parenchyma, XGP is more common in women and in 50-60 age group. Most cases are diffuse. E.coli and P.mirabils are the most common organisms in these patients with UTI

How well did you know this?
1
Not at all
2
3
4
5
Perfectly
8
Q
  1. In a patient who had a right nephrectomy ten years ago for RCC, a 3cm lesion in the cortex of the lower pole of the left kidney is evaluated with MRI. This appears isointenseon the in-phase sequence and low SI on the opposed phase sequence. No other renal lesions are demonstrated. The renal veins appear normal. What is the most likely diagnosis?

A. Oncocytoma

B. Simple cyst

C. RCC

D. AML

E. Metastatic deposit

A

D. AML

Demonstration of fat within a renal mass on CT or MRI is diagnostic of AML.

How well did you know this?
1
Not at all
2
3
4
5
Perfectly
9
Q
  1. A patient informs you she has a medical condition prior to pelvic MRI for evaluation of a pelvic malignancy. Which of the following would contraindicate buscopan injection?

A. Hypertension

B. Angina

C. CABG 3 years ago

D. Myasthenia gravis

E. Multiple sclerosis

A

D. Myasthenia gravis

Antimuscarinics are contraindicated in myasthenia gravis (but may be used to decrease the muscarinic side effects of anticholinesterases), paralytic ileus, pyloric stenosis and prostatic enlargement. They should be used with caution in Down’s syndrome, GORD, diarrhoea, ulcerative colitis, acute myocardial infarction, hypertension, conditions characterized by tachycardia (hyperthyroidism, cardiac insuffciency, cardiac surgery), pyrexia, pregnancy and in individuals susceptible to angle closure glaucoma. HBB improves image quality and lesion visualisation in oncologic pelvic MR.

How well did you know this?
1
Not at all
2
3
4
5
Perfectly
10
Q
  1. In dynamic renal imaging:

A. The patient must not eat or drink for 6 hours prior to the test

B. Provides information on total and divided function only

C. Total divided renal function are evaluated in addition to rates of transit through parenchyma and outflow track

D. Diethylene triamine pentaacetic acid (DTPA) has the advantage of higher renal concentration than inulin

E. DTPA is excreted by glomerular fltration and tubular excretion

A

C. Total divided renal function are evaluated in addition to rates of transit through parenchyma and outflow track

This investigation requires a hydrated patient to lie supine with knees slightly flexed to reduce lumbar lordosis.

DTPA is handled the same way as inulin.

D and E are correct for MAG3

How well did you know this?
1
Not at all
2
3
4
5
Perfectly
11
Q
  1. A 3cm cystic lesion is seen on CT. Thickened septae are noted, with nodular areas of calcifcation, with solid non-enhancing areas. Which of the following Bosniak classifcation best describes the lesion?

A. I

B. II

C. IIF

D. III

E. IV

A

D. III

Class II have at least one thin septa traversing them (< 1mm) and they have an appearance of thin areas of mural calcification or fluid content with greater attenuation.

These lesions are benign however IIF with numerous class II features should be followed up.

Class III features as above are indicative of malignancy and biopsy or surgical exploration is necessary.

Type IV cystic lesions are clearly malignant.

How well did you know this?
1
Not at all
2
3
4
5
Perfectly
12
Q
  1. Which of the following indicates T2 disease in renal cell carcinoma?

A. Perinephric fat involvement

B. Tumour size > 2.5cm

C. Tumour involvement of renal vein

D. Tumour involvement of adrenal gland

E. IVC involvement

A

B. Tumour size > 2.5cm

Stage T2 disease is indicated when the tumour size is greater than 2.5cm

How well did you know this?
1
Not at all
2
3
4
5
Perfectly
13
Q
  1. Regarding lymphoma of the kidneys:

A. Multiple focal nodules appear hyperdense on CT

B. Involved kidneys are usually atrophic

C. Focal masses appear high signal on T1

D. Focal masses appear hyperintense on T2

E. CT may demonstrate sheet like diffuse infltration of perirenal tissues

A

E. CT may demonstrate sheet like diffuse infltration of perirenal tissues

Diffuse infltration leads to renal enlargement. Focal lesions have a characteristic usually low attenuation post-contrast on CT, low SI on T1 and hypo-isointense on T2.

How well did you know this?
1
Not at all
2
3
4
5
Perfectly
14
Q
  1. Regarding metastatic sites of disease to the kidney:

A. Are frequently symptomatic

B. Usually occur from direct invasion

C. Haematogenous metastasis are usually > 3 cm

D. Are usually hypovascular on CT

E. Commonly calcify

A

D. Are usually hypovascular on CT

Metastases are usually small (< 3cm), multiple and confned to cortex.

The most common mode of spread is haematogenous.

Metastases tend not to invade the renal vein or calcify; they are more infltrative, less exophytic compared with renal cell carcinoma.

How well did you know this?
1
Not at all
2
3
4
5
Perfectly
15
Q
  1. Which of the following most favours RCC rather than TCC?

A. A central hypoechoic lesion on US

B. Intraluminal soft tissue mass in the calyx

C. Renal vein invasion

D. Infltration of renal sinus

E. Contrast outlining tumour in the pelvis

A

C. Renal vein invasion

In TCC inferior vena caval and renal vein invasion are uncommon

How well did you know this?
1
Not at all
2
3
4
5
Perfectly
16
Q
  1. Considering Squamous Cell Carcinoma (SCC) of the kidney:

A. Is the second most common tumour subtype affecting the kidney

B. Usually carries a better prognosis than RCC

C. Is usually indolent

D. Acute infection is involved in the aetiology

E. Renal calculi are present in most patients

A

E. Renal calculi are present in most patients

SCC of the kidney is a relatively rare condition. It carries a poor prognosis due to its aggressive nature. Both renal calculi and chronic infection have been implicated in its aetiology. Cross sectional imaging appearances are identical to those of TCC.

How well did you know this?
1
Not at all
2
3
4
5
Perfectly
17
Q
  1. Which is a cause of cortical rather than medullarynephrocalcinosis?

A. Acute cortical necrosis

B. Hyperoxaluria

C. Hypervitamosis D

D. Sarcoidosis

E. Renal tubular acidosis

A

A. Acute cortical necrosis

Acute cortical necrosis causes cortical nephrocalcinosis, whereas B-E are causes of medullary nephrocalcinosis

How well did you know this?
1
Not at all
2
3
4
5
Perfectly
18
Q
  1. Regarding hydronephrosis of pregnancy:

A. Left-sided dilated predominates

B. Occurs predominantly due to maternal hormones decreasing ureteric tone

C. Persists at most 3-4 days postpartum in most cases

D. Occurs in 90% of pregnant women by third trimester

E. Usually involves the entire length of the uterus

A

D. Occurs in 90% of pregnant women by third trimester

In most women dilatation disappears postpartum.

Resolution can take between a few days to several weeks.

Maternal hormones play a minor part.

Right-sided hydronephrosis is more common.

How well did you know this?
1
Not at all
2
3
4
5
Perfectly
19
Q
  1. Which is the most likely to cause bilateral small rather than large kidneys?

A. Medullary cystic disease

B. Multiple myeloma

C. PAN

D. Glycogen storage disease

E. Acute glomerulonephritis

A

A. Medullary cystic disease

Acute arterial hypotension, arteriosclerosis, nephrosclerosis and hereditary nephropathies including medullary cystic disease and Alport syndrome are all causes of bilateral small kidneys.

How well did you know this?
1
Not at all
2
3
4
5
Perfectly
20
Q

@# 47. A 50-year-old female undergoes CT for an echogenic lesion on ultrasound. Pre-contrast CT shows a lesion, which is well-defned and has increased attenuation (60HU). Postcontrast the lesion appears low in attenuation related to the surrounding parenchyma (61 HU). The diagnosis is:

A. Haemorrhagic renal cell carcinoma

B. Angiomyolipoma (AML) that has bled

C. Lymphoma

D. Haemorrhagic renal cyst

E. AML

A

D. Haemorrhagic renal cyst

A less than 10 HU increase post-contrast indicates benign hyperdense cyst. Other benign features include being sharply marginated and homogenous.

How well did you know this?
1
Not at all
2
3
4
5
Perfectly
21
Q

1) A patient who has no function in their native kidneys is found to have declining renal function 1 day after transplantation. A MAG3 renogram shows normal perfusion but diminished excretion. Which of the following processes is affecting the transplanted kidney?

a. acute rejection

b. chronic rejection

c. acute tubular necrosis

d. renal vein thrombosis

e. ciclosporin toxicity

A

c. acute tubular necrosis

Acute tubular necrosis is the commonest acute reversible cause of renal failure in the transplanted kidney and usually occurs within 24 hours. Of the complications of a transplanted kidney causing renal impairment, normal perfusion is seen in acute tubular necrosis, whereas renal vein thrombosis and transplant rejection have reduced perfusion accompanying the diminished excretion. Ciclosporin can cause a similar pattern of renal impairment but would be expected to occur 1 month after transplantation. Functional assessment of a transplanted kidney involves perfusion and excretion assessment with a MAG3 or DTPA renogram, MAG3 being the better test in transplant recipients with renal impairment. Doppler ultrasound resistive index measurement is also used, with a value of ,0.7 regarded as normal

How well did you know this?
1
Not at all
2
3
4
5
Perfectly
22
Q

2) A portal venous-phase CT of the abdomen and pelvis is performed in a 60-year-old man to investigate upper abdominal and back pain, which is attributed to features of pancreatitis on the scan. An incidental finding is of a rounded, renal lesion of diameter 3 cm, with average attenuation value of 80 HU and containing no significant component with a negative attenuation value on pixel densitometry. There are no previous images for comparison. What is the most likely diagnosis of the renal lesion?

a. angiomyolipoma

b. renal cell carcinoma

c. simple cyst

d. high-density cyst

e. infected cyst

A

b. renal cell carcinoma

A single portal venous phase CT is not the optimum image set to characterize renal parenchymal lesions. However, renal cell carcinoma is more commonly encountered than high-density cysts. Furthermore, carcinoma is most frequently found in men (2:1) aged over 50 years. Kidney neoplasms tend to have densities above 30 HU on an unenhanced CT and rise by more than 10–20 HU post-contrast, usually being above 70 HU in the portal phase.

How well did you know this?
1
Not at all
2
3
4
5
Perfectly
23
Q

4) A 55-year-old male has an ultrasound scan of the renal tract prompted by a single urinary tract infection. A kidney cyst of diameter 2 cm with a thin septum is seen. The septum has perceptible enhancement on CT. What is the most appropriate management from the choices below?

a. discharge with no follow-up

b. imaging follow-up

c. partial nephrectomy

d. nephrectomy

e. nephroureterectomy

A

b. imaging follow-up

An incidental, mildly complicated renal cyst has been uncovered. The Bosniak classification is a useful tool for evaluating cystic renal lesions, and guiding management. Simple cysts (Bosniak grade I) are thin walled, are of water density and have no enhancement. Minimally complicated cysts (grade II) may be clustered or septated, and have small curvilinear calcifications, a minimally irregular wall or high-density contents. Follow-up lesions (grade IIF) have perceptible enhancement of otherwise thin septations or are above 3cm in diameter with high-density contents. Surgical lesions (grade III) have thicker septa or walls, measurable enhancement, coarse irregular calcification and irregular margin, are multiloculated or can be a non-enhancing nodular mass. Clearly malignant lesions (grade IV) can have necrotic components, irregular wall thickening and enhancing solid elements.

How well did you know this?
1
Not at all
2
3
4
5
Perfectly
24
Q

9) CT scan of the chest, abdomen and pelvis is performed to stage a renal cell carcinoma. The tumour arises in, and is confined to, the upper pole of the left kidney with a maximum dimension of 5 cm. There is tumour thrombus in the left renal vein, inferior vena cava and right atrium. There are no enlarged lymph nodes and no metastases seen. According to the TNM classification what is the stage of the tumour?

a. T4 N0 M0

b. T2 N0 M0

c. T3a N0 M0

d. T3c N0 Mx

e. T3c N0 M0

A

e. T3c N0 M0

T1 and T2 renal cell carcinomas are limited to the kidney, and measure 7cm and .7cm respectively. T3 tumour extends beyond the kidney, into either the adrenal gland or perinephric tissues (T3a), the renal vein or vena cava below the diaphragm (T3b) or the vena cava above the diaphragm, or it invades the wall of the vena cava (T3c). T4 tumour invades beyond Gerota’s fascia. N1 or N2 nodal disease refers toinvolvement of a single regional node, or more than one regional node, respectively. Overall, T3c N0 M0 disease represents stage III disease.

How well did you know this?
1
Not at all
2
3
4
5
Perfectly
25
Q

10) A 30-year-old female has continued ipsilateral loin pain following surgery for pelviureteric junction obstruction. Ultrasound scan shows the renal pelvis to be less dilated than prior to surgery. MAG3 renogram is inconclusive with regard to the question of ongoing obstruction, and a Whitaker test is performed. The maximum pressure difference between the collecting system (antegrade) needle and the urinary catheter is found to be 10 cmH2O. What should the patient be advised regarding the findings of the Whitaker test?

a. this invasive test is also inconclusive

b. there is no evidence of pelviureteric junction or ureteric obstruction

c. there is ongoing pelviureteric junction obstruction

d. there is a second previously occult level of obstruction

e. the original diagnosis of pelviureteric junction obstruction is in doubt

A

b. there is no evidence of pelviureteric junction or ureteric obstruction

The Whitaker test is a pressure–flow study to evaluate ureteral obstruction or resistance in dilated non-refluxing upper tracts. Being invasive and time-consuming, it is usually performed only when excretory renograms are equivocal. A pressure difference of greater than 15cmH2O is abnormal.

How well did you know this?
1
Not at all
2
3
4
5
Perfectly
26
Q

15) An 80-year-old male has an IVU for unilateral loin pain. On the control film, the renal outline on the side of the pain is indistinct and enlarged. The same kidney has a staghorn calculus and shows no evidence of function following contrast injection. Which of the following is the most likely diagnosis?

a. xanthogranulomatous pyelonephritis

b. renal tuberculosis

c. hyperparathyroidism

d. hydatid cyst

e. carcinoma

A

a. xanthogranulomatous pyelonephritis

On the IVU, the features of xanthogranulomatous pyelonephritis are unilateral reniform enlargement, ipsilateral renal hypofunction and nephrolithiasis. The condition is probably produced by chronic lowgrade obstruction and chronic bacteriuria. Renal parenchyma is replaced by lipid-laden histiocytes causing renal expansion. This expansion can cause ‘stone fracture’ with obvious separation of the fracture fragments on plain film. Renal tube==rculosis causes dystrophic calcification that can be nodular, curvilinear or amorphous. The calcification is typically multifocal, involving other sites of the urinary tract, and there may be kidney scarring. Hyperparathyroidism causes medullary calcification. Six per cent of renal carcinomas have amorphous, irregular or occasionally curvilinear calcification, while hydatid cysts exhibit curvilinear or heterogeneous calcification

How well did you know this?
1
Not at all
2
3
4
5
Perfectly
27
Q

17) A 30-year-old male is investigated by renal tract ultrasound scan for renal impairment. Both kidneys are smooth in outline but enlarged. Which of the following diagnoses typically produces this pattern of renal enlargement?

a. autosomal dominant polycystic kidney disease

b. von Hippel–Lindau disease

c. sickle cell disease

d. metastases

e. nephroblastomatosis

A

c. sickle cell disease

Causes of bilateral smooth renal enlargement include diabetic nephropathy, acute glomerulonephritis, collagen vascular disease, vasculitis, AIDS nephropathy, leukaemia, lymphoma, autosomal recessive polycystic renal disease, acute interstitial nephritis, sickle cell disease, thalassaemia, acromegaly, amyloidosis, myeloma and acute urate nephropathy. When the bilateral renal enlargement is caused by masses, the differential diagnosis includes autosomal dominant polycystic disease, acquired renal cystic disease, lymphoma, metastases, Wilms’ tumours, tuberous sclerosis, von Hippel–Lindau syndrome, multiple oncocytomas and nephroblastomatosis

How well did you know this?
1
Not at all
2
3
4
5
Perfectly
28
Q

26) A 35-year-old female has investigations for episodic right loin pain. Ultrasound scan of the renal tract is unremarkable. A DMSA scan is performed with the patient sitting, and shows only 30% contribution to the total tracer activity from the right kidney. When the counts are repeated supine, the contribution from the right kidney is 50%. What is the most likely abnormality of the right kidney?

a. nutcracker kidney

b. nephroptosis

c. pelviureteric junction obstruction

d. ureteric calculus

e. vesicoureteric reflux

A

b. nephroptosis

Ptosis of the mobile kidney when erect can cause symptoms and underestimation of parenchymal DMSA uptake. Since the differential function may be a factor in considering removal of a kidney, the technique should account for the possibility of nephroptosis influencing the counts. A nutcracker kidney is a rare cause of left-sided loin pain and haematuria; it is caused by compression of the left renal vein between the aorta and superior mesenteric artery.

How well did you know this?
1
Not at all
2
3
4
5
Perfectly
29
Q

27) An 80-year-old male with a history of nephrectomy for renal cell carcinoma is found at follow-up to have a heterogeneously enhancing 25 mm lesion confined to his remaining kidney. No enlarged nodes or metastases are present. The lesion is biopsied and found to be an adenocarcinoma. The patient decides upon radiofrequency ablation as treatment. A CT scan 1 month after the ablation quantifies the average post-contrast enhancement of the tumour as 8 HU. Which of the following best represents the degree of success of the radiofrequency ablation?

a. failed

b. residual enhancing tumour requiring repeat ablation

c. residual enhancing tumour but no value in repeat ablation

d. successful

e. indeterminate

A

d. successful

The practice of radiofrequency ablation of renal tumours is emerging. Currently, CT 1 month after the procedure is used to assess treatment success. If enhancing prior to ablation, the tumour is regarded as ablated if there is ,10 HU rise in attenuation following contrast administration. Bulky persistent irregular peripheral enhancement is the commonest appearance of an incompletely treated lesion

How well did you know this?
1
Not at all
2
3
4
5
Perfectly
30
Q

30) A 65-year-old male being investigated for microscopic haematuria has an ultrasound scan, which suggests a 20 mm tumour in the cortex of the interpolar region of the left kidney. CT scan confirms an enhancing mass in the same location. On DMSA SPECT, this abnormality has good uptake. Which of the following is the most appropriate management?

a. no further action

b. biopsy

c. nephrectomy

d. image-guided drainage

e. chemotherapy

A

a. no further action

The abnormality described is prominent or hypertrophic cortex since it takes up DMSA, which in the kidneys is a parenchymal tracer. Renal cell carcinoma, cysts, abscess, haematoma, scar and infarct would be seen as photopenic areas on DMSA SPECT, if large enough.

How well did you know this?
1
Not at all
2
3
4
5
Perfectly
31
Q

34) A 35-year-old male with autosomal dominant polycystic kidney disease has been shown on CT to have, among the innumerable renal cysts, several high-density cysts. Which of the following MRI sequences would be most useful in detecting a renal cell carcinoma among haemorrhagic or proteinaceous cysts?

a. T1W

b. T2W

c. T1W post-gadolinium

d. T1W fat-suppressed post-gadolinium

e. T1W post-gadolinium with pre-contrast T1W signal subtracted

A

e. T1W post-gadolinium with pre-contrast T1W signal subtracted

The cornerstone of diagnosis here is the post-contrast enhancement of renal cell carcinoma. To identify this among the high T1 signal of haemorrhage or protein within cysts, it is ideal to subtract the precontrast T1 signal. Fat suppression will not remove the distracting high signal from mildly complicated cysts. Risk of renal cell carcinoma is increased in adult polycystic renal disease when in renal failure. Hence caution may be required since at certain levels of renal impairment the use of MRI contrast is not advised

How well did you know this?
1
Not at all
2
3
4
5
Perfectly
32
Q

38) An adult male patient who has been taking over-the-counter analgesics regularly for years has an IVU for ureteric colic. No radioopaque calculi are seen on the control film. With contrast in the renal calyces, they are noted to be club shaped on the side of the pain. On the same image, there is a triangular filling defect in the renal pelvis. The colic is most likely to be caused by which of the following?

a. radio-opaque stone

b. radiolucent stone

c. sloughed papilla

d. blood clot

e. transitional cell carcinoma

A

c. sloughed papilla

Predisposing factors to papillary necrosis include diabetes mellitus, analgesic nephropathy/abuse, sickle cell disease, pyelonephritis, obstructive uropathy, tuberculosis, trauma, cirrhosis, coagulopathy and renal vein thrombosis. None of the other options easily explains the club-shaped calyx. Blood clots from renal or tumour haemorrhage do cause ureteric colic but tend to elongate along the pelvis and ureter

How well did you know this?
1
Not at all
2
3
4
5
Perfectly
33
Q

40) A 38-year-old man presents with a classic history of ureteric colic. Plain abdominal film is unremarkable, and CT KUB shows ureteric dilatation and periureteric stranding down to the vesicoureteric junction on the side of the pain. No radio-opaque calculi are seen on the CT scan. Ultrasound examination shows a tiny, densely echogenic focus within the bladder wall, at the same vesicoureteric junction. For which of the following conditions is the patient most likely to be receiving treatment?

a. diabetes mellitus

b. asthma

c. HIV

d. gastro-oesophageal reflux

e. headaches

A

c. HIV

While around 10% of renal and ureteric stones are radiolucent on plain film, almost all are opaque on CT. One exception is the tiny radiolucent calculi formed in patients on protease inhibitors such as indinavir used in the treatment of HIV/AIDS

How well did you know this?
1
Not at all
2
3
4
5
Perfectly
34
Q

50) A 60-year-old male has an ultrasound scan of the renal tract for renal colic. There is an echo-free, thin-walled structure in the renal sinus with posterior acoustic enhancement and dilatation of the major calyces. A CT KUB (unenhanced) does not add to this appearance, but a 10-minute delayed, contrast-enhanced CT shows that the calyces are obstructed while the renal pelvis is not dilated but stretched over the non-enhancing sinus abnormality. What is the most likely diagnosis?

a. sinus lipomatosis

b. parapelvic cyst

c. pelviureteric junction obstruction

d. transitional call carcinoma

e. renal cell carcinoma

A

b. parapelvic cyst

The main differential diagnosis for a parapelvic or renal sinus cyst is hydronephrosis. Such cysts may present with pain due to obstructive caliectasis, but rarely cause hydronephrosis. They are found in 1.5% of autopsies and represent 4–6% of all renal cysts. A distinction is made in this question between renal and ureteric colic, the former symptom located to the loin and the latter more typically being loin to groin.

How well did you know this?
1
Not at all
2
3
4
5
Perfectly
35
Q

71) A 60-year-old female who has declined intervention for a renal angiomyolipoma of diameter 6 cm presents with flank pain, hypotension and tachycardia. In this scenario, which of the following is likely to account for the presentation?

a. torsion of the angiomyolipoma

b. haemorrhage from the angiomyolipoma

c. rupture of the angiomyolipoma

d. leak from the non-aneurysmal abdominal aorta

e. nephroptosis

A

b. haemorrhage from the angiomyolipoma

Angiomyolipomas are benign hamartomatous tumours that can occur in the kidneys. They contain fat, smooth muscle and abnormal blood vessels. Eighty per cent are sporadic and occur most often in females aged 50–80 years. Twenty per cent of patients with angiomyolipomas have tuberous sclerosis. Retroperitoneal bleed is the most significant complication and can be catastrophic. The risk increases with size of the lesion due to increased abnormal vasculature. Haemorrhage occurs because of large tortuous vessels and aneurysms. Embolization is performed for symptomatic (flank pain) angiomyolipomas, those that have bled at any size, and prophylactically when over 4cm.

How well did you know this?
1
Not at all
2
3
4
5
Perfectly
36
Q

81) A 70-year-old man with previously diagnosed, bilateral renal calculi is seen with fever, rigors and left loin pain. Ultrasound scan shows a dilated left renal collecting system. Which of the following is the most management?

a. oral antibiotics with outpatient follow-up

b. urinary catheter, intravenous antibiotics and admission to hospital

c. left nephrostomy and antibiotics

d. bilateral nephrostomy

e. intramuscular anti-spasmodics

A

c. left nephrostomy and antibiotics

Pyonephrosis secondary to an obstructing calculus is the likely diagnosis. Percutaneous nephrostomy is indicated for temporary or permanent relief of an obstructed urinary system (malignant or benign obstructive uropathy), pyonephrosis, renal stones, iatrogenic ureteric injury, transplant kidney ureteric obstruction, and vesicovaginal fistula. A ‘onestab’ puncture technique or Seldinger technique can be used. For any procedures where urinary infection is suspected or in stone disease, prophylactic antibiotics are mandatory – for example, 80mg gentamicin or 750mg cefuroxime. Clotting must also be checked and if necessary corrected. The major complication rate is around 3% while minor complications occur in around 15%. Major complications are septicaemia, blood loss requiring transfusion, pleural or abdominal viscera puncture, or transcolonic approach. Minor complications include retroperitoneal urine extravasation, clot colic from macroscopic haematuria and tube complications. Tube complications include catheter dislodgement, blockage, leaking, kinking and fracture.

How well did you know this?
1
Not at all
2
3
4
5
Perfectly
37
Q

84) A 60-year-old man has a 4 cm rounded mass arising within the right kidney. It has heterogeneous, strong post-contrast enhancement. Calcification is also evident within the tumour. Which of the following features of this renal mass would favour the diagnosis of renal cell carcinoma over angiomyolipoma?

a. marked vascularity

b. calcification

c. fat within the tumour

d. round morphology

e. hyperechogenic on ultrasound scan

A

b. calcification

Angiomyolipoma should not have calcification whereas it is seen in 10% of renal cell carcinomas. Both these tumours can be hypervascular. The cornerstone of diagnosis of an angiomyolipoma is identifying fat on CT or MRI; however, fat has been reported in renal cell carcinoma, and peripheral or renal sinus fat can become trapped in any large renal tumour

How well did you know this?
1
Not at all
2
3
4
5
Perfectly
38
Q

95) A portal venous phase abdominal CT scan of a 65-year-old man demonstrates an ill-defined, rounded area 4 cm in diameter within a kidney. It is heterogeneous but predominantly of attenuation value above 70 HU. It contains small dense calcific foci. Which additional feature suggests that the lesion is more likely to be a renal cell carcinoma than a transitional cell carcinoma of the collecting system?

a. thickened indurated pelvicalyceal wall

b. central location of the tumour with centrifugal expansion that compresses renal sinus fat

c. renal parenchymal invasion with renal contour preservation

d. renal vein thrombus

e. further mass arising from the urinary bladder wall

A

d. renal vein thrombus

Differentiation of renal cell from transitional cell carcinoma is helpful for planning surgical treatment since transitional cell carcinoma of the renal collecting system requires the more extensive surgical procedure of nephroureterectomy. Renal vein thrombus is seen with renal cell carcinoma while all other options given are features of transitional cell carcinoma of the kidney. Delayed contrast CToffers a pyelographic phase on which collecting system, ureter and bladder filling defects are clearly demonstrated. A urothelial field effect can occur, resulting in multiple transitional cell carcinomas throughout the renal tract. Renal cell carcinoma, as it expands, tends to distort the renal outline and is more likely to be peripheral and exophytic

How well did you know this?
1
Not at all
2
3
4
5
Perfectly
39
Q

94) A diabetic patient with long-standing mild renal impairment requires an angiogram, and it is decided that iodinated contrast will be used. Which of the following is most likely to prevent the patient from developing contrast-induced nephropathy?

a. prior administration of acetylcysteine

b. thorough hydration of the patient

c. oral fluid restriction

d. concurrent diuretic administration

e. use of high-osmolar contrast medium

A

b. thorough hydration of the patient

A significant contribution to the evidence base used to guide practice in contrast-induced nephropathy (CIN) comes from the NEPHRIC study group. The trial was a randomized, prospective, double-blind, multicentre study performed in 17 centres in Denmark, France, Germany, Spain and Sweden, and consisted of 129 patients. CIN is acute renal impairment with an absolute increase in serum creatinine of at least 0.5mg/dl (44.2 mmol/l) or a relative increase of at least 25% from baseline. A rise of 1mg/dl is less frequently used as the definition. CIN usually peaks on day 2 or 3 following iodinated contrast injection, with a return to baseline within 2 weeks. Return to baseline is not always seen. Low-osmolar iodinated contrast media have a low rate of CIN in the general population – less than 2%. In patients with increased risk of CIN due to diabetes mellitus or pre-existing renal impairment, this rate rises significantly. It has also been shown that low-osmolar contrast causes less CIN than high-osmolar (order of 1800 mosmol/kg water) contrast in high-risk patient groups. No difference in CIN rate is observed in lowrisk groups when iso-osmolar is compared with low-osmolar contrast agents. The NEPHRIC study group shows that in people with diabetes or those with renal impairment having iliofemoral or coronary angiography there is a reduced rate of CIN when using iso-osmolar iodinated contrast as opposed to low-osmolar contrast medium. Osmotic diuresis causing increased sodium load to the distal nephron, with consequent increased medullary work, possible hypoxia and volume depletion giving rise to activation of vasoregulatory hormone systems, is suggested as the reason for the findings. Vigorous hydration is encouraged as perhaps the most important measure to try to avoid CIN. There is evidence both supporting and rejecting the nephroprotective effect of the free radical scavenger acetylcysteine when given before the iodinated contrast media

How well did you know this?
1
Not at all
2
3
4
5
Perfectly
40
Q

96) An adult male is initially investigated for abnormal liver function tests. Eventually, the diagnosis of Stauffer’s syndrome is pronounced. What are the likely CT findings?

a. liver mass in keeping with hepatocellular carcinoma with renal metastases

b. renal mass in keeping with renal cell carcinoma with liver metastases

c. renal mass in keeping with renal cell carcinoma and hepatosplenomegaly without focal hepatic or splenic lesions

d. hepatosplenomegaly and bilateral renal enlargement without focal lesions in any of these organs

e. renal mass in keeping with renal cell carcinoma with a pancreatic head metastasis

A

c. renal mass in keeping with renal cell carcinoma and hepatosplenomegaly without focal hepatic or splenic lesions

Stauffer described a syndrome of nephrogenic hepatopathy in which a renal cancer without liver metastases causes hepatosplenomegaly and abnormal liver function. Renal cell carcinoma paraneoplastic phenomena include erythrocytosis and hypercalcaemia.

How well did you know this?
1
Not at all
2
3
4
5
Perfectly
41
Q

97) A clinical trial of a novel chemotherapy agent for renal cell carcinoma is being undertaken. The response of the primary tumour and nodal and distant metastases will be assessed according to the RECIST criteria. On axial imaging, the long axis of every lesion present on the pre-treatment scan has decreased on the post-treatment CT. However, the patient has progressive disease. Which additional feature from the list below would explain this?

a. the improvement in the summed long axes of five lesions is not more than 30%

b. a new site of disease is identified

c. the lesions have not all disappeared

d. sustained improvement was not proven by a repeat CT at 4 weeks

e. the improvement in the summed products of bidimensional measurements is not more than 50%

A

b. a new site of disease is identified

Two criteria are commonly used for assessing cancer response to treatment: WHO and RECIST. The latter stands for Response Evaluation Criteria in Solid Tumours, and it is this tool that is usually used in treatment trials. The WHO criteria compare a summed area product (longest axial dimension multiplied by the longest axial dimension perpendicular to this). RECIST sums the longest axial dimension and compares this across scans. Complete response for both criteria is disappearance of disease, confirmed at 4 weeks. A partial response according to the WHO is 50% or more reduction in the summed area product following treatment and confirmed at 4 weeks. RECIST requires a 30% or greater decrease in the summed longest diameters, confirmed at 4 weeks. Progressive disease is defined as a 25% increase in summed area product for the WHO criteria and a 20% increase in summed longest diameter according to RECIST. Progressive disease also results from the appearance of any new site of disease. Stable disease reflects changes of magnitude that do not achieve partial response or progressive disease

How well did you know this?
1
Not at all
2
3
4
5
Perfectly
42
Q

5 A 28 year old man presents with loin pain and dipstick positive haematuria. A CT KUB is arranged for further investigation. Which of the following statements is not correct regarding this investigation?

(a) It is now the initial investigation of choice

(b) lndinavir related calculi are not well seen

(c) Urate caltuli are well visualised

(d) Nephrocalcinosis may result in a false positive examination

(e) The ‘comet tail’ sign confirms a ureteric calculus rather than phlebolith

A

(e) The ‘comet tail’ sign confirms a ureteric calculus rather than phlebolith

lndinavir and pure matrix calculi are the only types of stone not well visualized on CTKUB. The ‘soft tissue rim’ sign refers to circumferential thickening of the ureteric wall around a calculus as opposed to the ‘comet tail’ sign which is seen around phleboliths.

How well did you know this?
1
Not at all
2
3
4
5
Perfectly
43
Q

11 A 30 year old man presents with bilateral loin pain. KUB shows coarse granular calcification widely distributed in the region of the renal pyramids. US shows increased echogenicity of the renal pyramids with some posterior acoustic shadowing. Which of the following is least likely?

(a) Alport syndrome

(b) Medullary sponge kidney

(c) Milk-alkalr syndrome

(d) Hyperparathyroidism

(e) Renal tubular acidosis

A

(a) Alport syndrome

Medullary nephrocalcinosis has a wide differential diagnosis, but options (b), (d) and (e) account for 70% of cases. Other causes include papillary necrosis and drugs such as Frusemide. Alport syndrome (hereditary chronic nephritis) typically gives rise to cortical calcificatio

How well did you know this?
1
Not at all
2
3
4
5
Perfectly
44
Q

13 A 60 year old man presents with biliary colic. At US an incidental finding of a well-demarcated 5 cm mass of low echogenicity is noted arising from the right kidney. CT confirms a renal mass with a central low attenuation scar. MRI shows the mass to be hypointense on T1W and hyperintense on T2W with enhancement after i.v. gadolinium administration, although the central scar enhances less well than the remainder of the mass. What is the likeliest diagnosis?

(a) Renal cell carcinoma

(b) Oncocytoma

(c) Transitional cell carcinoma

(d) Hamartoma

(e) Metastasis

A

(b) Oncocytoma

In addition to the above findings, a ‘spoke-wheel’ appearance at angiography and a photopaenic area on 99mrc-DMSA scan may be seen.

How well did you know this?
1
Not at all
2
3
4
5
Perfectly
45
Q

20 A patient is diagnosed with a 4 cm right upper pole renal cell carcinoma. Staging investigations demonstrate tumour thrombus in the renal vein extending into the IVC, but no local lymphadenopathy and no evidence of distant metastases. What is the Robson staging of this tumour?

(a) Stage II

(b) Stage Illa

(c) Stage lllb

(d) Stage Ille

(e) Stage IV

A

(b) Stage Illa

Stage I includes tumours that are confined entirely to the kidney. Stage II tumours invade the perinephric fat or adrenal gland on the same side. Stage Illa tumours extend into the renal vein or the IVC. Stage lllb tumours involve local LNs. Stage Illc combines stages Illa and lllb. Stage IV tumours describes those with distant metastases.?

How well did you know this?
1
Not at all
2
3
4
5
Perfectly
46
Q

21 A 24 year old with known ureteric reflux disease in childhood presents with loin pain. KUB shows an extensive calculus involving the lower pole and interpolar calyces. What is the likeliest composition of the stone?

(a) Magnesium ammonium phosphate

(b) Xanthine

(c) Cysteine

(d) Urate

(e) Calcium oxalate

A

(a) Magnesium ammonium phosphate

70% of staghorn calculi are composed of magnesium ammonium phosphate (struvite stones). The remainder are cysteine or urate stones. These large calculi are often seen in patients with a history of recurrent infections.

How well did you know this?
1
Not at all
2
3
4
5
Perfectly
47
Q

34 With regards to a 99mTc-MAG3 renogram, which of the following statements is incorrect?

(a) The pure blood flow phase lasts for 2 minutes

(b) The uptake phase should be measured 60-120 secs after tracer administration

(c) The peak of the curve represents the maximum activity in the kidney .

(d) Frusemitle should be administered in the presence of dilatation.

(e) A renal tumour would cause a photopaenic area within the kidney

A

(a) The pure blood flow phase lasts for 2 minutes

The pure blood flow phase is the first phase of the examination and lasts up to 40 secs (equivalent to an arterial phase study).

How well did you know this?
1
Not at all
2
3
4
5
Perfectly
48
Q

45 A 35 year old woman presents with a history of loin pain. CT shows a large perinephric haematoma. She cannot recall any significant trauma other than whilst playing with her child. Which of the following is the least likely cause?

(a) Multicystic dysplastic kidney

(b) Renal cell carcinoma

(c) Autosomal dominant polycystic kidney disease

(d) Polyarteritis nodosa

(e) Angiomyolipoma of the kidney

A

(a) Multicystic dysplastic kidney

Occult RCC, polyarteritis nodosa, ADPKD and angiomyolipoma are well recognized causes of perinephric haematoma after innocuous trauma. MCDK do not typically bleed externally into the perinephric space.

How well did you know this?
1
Not at all
2
3
4
5
Perfectly
49
Q

47 A 32 year old woman undergoes an IVU which demonstrates bilateral striated nephrograms. Which of the following is least likely?

(a) Tamm-Horsfell proteinuria

(b) Hypotension

(c) Amyloidosis

(d) Acute pyelonephritis

(e) Medullary sponge kidney

A

(c) Amyloidosis

Striated nephrograms are transiently seen in acute extra-renal obstruction along with hypotension and intratubular obstruction. Other causes of bilateral striated nephrograms include rhabdomyolysis and cystic renal disease. Causes of a (usually) single striated nephrogram include renal contusion and renal vein thrombosis

How well did you know this?
1
Not at all
2
3
4
5
Perfectly
50
Q

49 A 50 year old man is found to have an incidental renal lesion. On unenhanced CT, the lesion is 2 cm, round, homogeneous, well defined, without calcification and situated in the upper pole of the right kidney. It has an attenuation of 60 Hounsfield units, which after the administration of intravenous contrast medium increases to 69 HU. Which of the following is the likeliest diagnosis?

(a) Renal cell carcinoma

(b) Lipid poor angiomyolipoma

(c) Renal oncocytoma

(d) Simple cyst

(e) Metanephric adenoma

A

(d) Simple cyst

Although most cysts are of low attenuation, by far the commonest cause of an otherwise benign appearing, non-enhancing (<10 HU increase), hyperattenuating (normal renal parenchyma approximately 40 HU on unenhanced CT) lesion is a hyperdense cyst containing proteinaceous material. Lipid poor angiomyolipomas are a rare (3-4%) subset of angiomyolipomas.

How well did you know this?
1
Not at all
2
3
4
5
Perfectly
51
Q

@#e 1 A 52 year old man presents with loin pain and microscopic haematuria. KUB reveals no abnormality. A CT KUB demonstrates a 7 mm mid-ureteric calculus. What is the most likely composition of the calculus?

(a) Calcium phosphate

(b) Calcium oxalate

(c) Urate

(d) Xanthine

(e) Cystine

A

(c) Urate

Although both xanthine and urate stones are radiolucent, urate stones are much more common. Cystine stones are mildly opaque. Calcium oxalate and calcium phosphate stones are radio-opaque

How well did you know this?
1
Not at all
2
3
4
5
Perfectly
52
Q

5 A frail 72 year old diabetic woman has serum creatinine measurements which are persistently at the upper end of the normal range. A nuclear medicine clearance scan is requested for a more accurate estimate of glomerular filtration rate. Which of the following tracers would be most appropriate?

(a) 51 Cr-EDTA

(b) 99mTc-DMSA

(c) 99mTc-Glucoheptonate

(d) 99mTc-HIDA

(e) 99mTc-MAG3

A

(a) 51 Cr-EDTA

GFR is a commonly accepted standard measure of renal function. It can be measured by tracers that are cleared exclusively by glomerular filtration, the most common being 51 Cr-EDTA (the.standard radiopharmaceutical in Europe) and 99mTc-DTPA (more common in the USA).

How well did you know this?
1
Not at all
2
3
4
5
Perfectly
53
Q

8 A 52 year old man is referred for a renal US as part of an investigation for proteinuria. US shows a 2 cm round, well-defined right upper pole cystic lesion. At CT, the lesion demonstrates internal hairline-thin septae and barely perceptible enhancement after the administration ofintravenous contrast medium. Which of the following statements is true?

(a) This is a Bosniak IV lesion

(b) This lesion should be surgically excised

(c) This lesion should be followed-up

(d) This is a Bosniak I lesion

(e) Bosniak classification can be defined on US

A

(c) This lesion should be followed-up

The Bosniak classification is a CT classification of cystic renal lesions. Bosniak I are simple cysts, Bosniak II are minimally complicated cysts. Bosniak llf (lesionE1 that should be followed-up) have hairline thin septae and subtle wall enhancement or are intrarenal lesions >3 cm with high density content. Bosniak Ill cysts are complicated lesions with irregular thickened septae, measurable enhancement and coarse irregular calcification. These should be treated surgically. Bosniak IV lesions are clearly malignant with a large necrotic component, irregular wall thickening and solid enhancing elements.

How well did you know this?
1
Not at all
2
3
4
5
Perfectly
54
Q

13 Which of the following are not radiological signs of renal artery stenosis?

(a) Delayed appearance of contrast material on IVU

(b) Notching of proximal ureter on IVU

(c) Spectral broadening and flow reversal on Duplex US

(d) Decreased density of contrast material on IVU

(e) Tardus/ parvus waveform on Duplex US

A

(d) Decreased density of contrast material on IVU

An increase in density of contrast material is seen on IVU due to relative increased water reabsorption. Other IVU signs include delayed washout of contrast medium and lack of distension of the collecting system. US findings include peak Renal artery:Aortic velocity >3.5, absence of blood flow during diastole, no detectable Doppler signal in a visualized renal artery, loss of early systolic peak and a resistive index <0.56.

How well did you know this?
1
Not at all
2
3
4
5
Perfectly
55
Q

16 A 63 year old man with a history of renal calculi presents with a 4 month history of unilateral flank pain, weight loss and fever. Renal US shows loss of cortico-medullary differentiation, with hypoechoic dilated calyces with an echogenic rim and multiple hypoechoic masses with low level internal echoes replacing much of the renal parenchyma. CT confirms a 1 cm calculus in the renal pelvis and multiple lowattenuation masses within the kidney. Angiography shows displacement of segmental arteries around an avascular masses. What is the likeliest diagnosis?

(a) Candida pyelonephritis

(b) Renal sarcoma

(c) Chronic renal infarction

(d) Multilocular renal cell carcinoma

(e) Xanthogranulomatous pyelonephritis

A

(e) Xanthogranulomatous pyelonephritis

XGP is a chronic granulomatous infection in chronic renal obstruction with progressive macrophage infiltration of the renal parenchyma. Fungal renal infections can cause pyelonephritis with fungal balls. Renal sarcoma typically presents with an infiltrative and expansile mass. Mutilocular RCC is a type of a cystic RCC

How well did you know this?
1
Not at all
2
3
4
5
Perfectly
56
Q

17 A 50 year old metformin-controlled diabetic man presents with renal colic. His serum creatinine is 90 μmol/L. Initial CT KUB is equivocal and the decision is made to administer i.v. contrast medium. Which of the following is incorrect?

(a) Contrast induced nephropathy (CIN) is usually transient

(b) The single most important risk factor for CIN is pre-test GFR

(c) High osmolar contrast media are more nephrotoxic than low osmolar contrast media

(d) The patient should stop his metformin for 48 hours after the examination

(e) The patient should be well hydrated prior to the examination

A

(d) The patient should stop his metformin for 48 hours after the examination

If the serum creatinine is within the normal range, or the estimated GFR is >60 ml/min, then metformin need not be stopped. If these are abnormal then discussion with the clinical team is required.

How well did you know this?
1
Not at all
2
3
4
5
Perfectly
57
Q

23 A 50 year old woman develops microscopic haematuria and is referred for an IVU. This demonstrates bilateral bulbous cavitation of the papillae with streaks of contrast material extending from the fornix parallel to the axis of the papillae and diminished density of the nephrogram. In addition, small filling defects are seen in the renal calyces. Which of the following is most likely to account for these findings?

(a) Sickle cell disease

(b) Analgeslc nephropathy

(c) Renal vein thrombosis

(d) Christmas disease

(e) Hepatic cirrhosis

A

(b) Analgeslc nephropathy

These are the IVU signs of papillary necrosis. There is a wide differential diagnosis for these appearances including diabetes mellitus, SCD, obstructive uropathy, and pyelonephritis. Overall, diabetes is the most common cause, but analgesic nephropathy is particularly common in middle-aged females.

How well did you know this?
1
Not at all
2
3
4
5
Perfectly
58
Q

24 A 35 year old man presents with night sweats and weight loss. CT examination shows multiple enlarged lymph nodes in the thorax and abdomen with homogeneous, ill defined masses in the kidney. A diagnosis of renal lymphoma is suspected. Which of the following statements is incorrect?

(a) Renal involvement is more common in NHL than Hodgkin’s disease

(b) Primary renal lymphoma is less common than secondary renal lymphoma

(c) It is more commonly bilateral than unilateral

(d) Enhanced through transmission is commonly seen on US

(e) Avid enhancement after contrast medium administration is seen on CT

A

(e) Avid enhancement after contrast medium administration is seen on CT

Renal lymphoma typically occurs in B-Cell non-Hodgkin’s lymphoma, where it is the second most common organ involved after the haematopoietic and reticulo-endothelial systems. Primary renal lymphoma is uncommon as the normal kidney does not contain significant amounts of lymphoid tissue. Intra-renal lymphoma appears as an ill-defined, homogeneous mass on CT which enhance poorly relative to the surrounding parenel

How well did you know this?
1
Not at all
2
3
4
5
Perfectly
59
Q

26 A 30 year old woman presents to the emergency department hypotensive and unwell. CT shows retroperitoneal blood which is arising from a 6 cm renal mass. The mass is well defined and on unenhanced imaging has a mean density of -5 HU. What is the most likely diagnosis?

(a) Renal oncocytoma

(b) Renal cell carcinoma

(c) Renal angiomyolipoma

(d) Renal liposarcoma

(e) Renal lipoma

A

(c) Renal angiomyolipoma

Renal AML is a benign tumour containing fat, smooth muscle and thick-walled blood vessels. They can bleed (50-60% of AMLs > 4cm bleed spontaneously), and can cause haemorrhagic shock (Wunderlich syndrome). Fat on non-contrast CT in the absence of calcification is virtually diagnostic. They can be associated with tuberous sclerosis (in 80% of TS patients), VHL, and NF

How well did you know this?
1
Not at all
2
3
4
5
Perfectly
60
Q

32 A 26 year old man is involved in an RTA, and falls from his motorcycle at 60 mph. On arrival in the emergency department he is alert and mobile with a blood pressure of 115/72 and a pulse rate of 60/min. He complains of mild discomfort over his left lower rib cage. A urine sample demonstrates rose coloured urine. Which of the following is the most appropriate next step?

(a) Renal ultrasound

(b) Admit for observation and image should he deteriorate

(c) Contrast enhanced CT of the abdomen

(d) IVU

(e) Renal angiography

A

(c) Contrast enhanced CT of the abdomen

With a high velocity injury and frank haematuria, CT is the investigation of choice. 25% of patients with frank haematuria and blunt trauma have a significant renal injury. Normotensive patients with microscopic haematuria have a significant renal injury in <0.2%.

How well did you know this?
1
Not at all
2
3
4
5
Perfectly
61
Q

40 A 54 year old man presents with microscopic haematuria. CT shows a 3 cm renal mass containing flecks of calcification and which demonstrates heterogeneous enhancement with i.v. contrast medium. What is the likeliest diagnosis?

(a) Oncocytoma

(b) Angiomyolipoma

(c) Lymphoma

(d) Transitional cell carcinoma

(e) Renal cell carcinoma

A

(e) Renal cell carcinoma

RCC is by far the commonest malignant renal tumour. 10% of these tumours calcify

How well did you know this?
1
Not at all
2
3
4
5
Perfectly
62
Q

41 Regarding the anatomy of the normal kidney, which of the following statements is true?

(a) The renal arteries typically have three divisions

(b) The column of Bertin is isoechoic to renal cortex

(c) The column of Bertin represents the fusion of the embryological anterior and posterior kidneys

(d) The right renal artery typically arises at the level of the L3 intervertebral disc

(e) The left renal artery is usually longer and lower than the left

A

(b) The column of Bertin is isoechoic to renal cortex

The column of Bertin is an area of focal cortical hyperplasia, typically seen between the upper and interpolar region. It has the imaging characteristic of renal cortex. Its main clinical significance is that it can be mistaken for a renal tumour. Both renal arteries usually have two divisions. The renal arteries typically arise at the L1/L2 level. The right renal artery is usually longer and lower than the left.

How well did you know this?
1
Not at all
2
3
4
5
Perfectly
63
Q

43 With regards to 99mTc-DTPA in renal imaging, which of the following statements is incorrect?

(a) It has a rapid extravascular, extracellular distribution

(b) It is cleared mainly by tubular secretion

(c) It undergoes 5-10% plasma protein binding

(d) Good post test hydration and frequent voiding reduce radiation dose

(e) It is useful in providing information relating to GFR

A

(b) It is cleared mainly by tubular secretion

It is mainly cleared by glomerular filtration hence its utility as a GFR agent. It is useful in assessing the degree of obstructive uropathy. 99mTc-MAG3 is cleared by tubular secretion and is the agent of choice in patients with renal insufficiency as it is not GFR dependent.

How well did you know this?
1
Not at all
2
3
4
5
Perfectly
64
Q

61 A 3 year old child born to an HIV positive mother is found to have bilateral enlarged kidneys on ultrasound. Which of the following conditions is least likely?

(a) ADPKD

(b) Obstructive pelvic mass

(c) HIV nephropathy

(d) Medullary cystic disease

(e) Lymphoma

A

(d) Medullary cystic disease

Medullary cystic disease appears either as a juvenile (rapidly progressive) or as an infantile form. It is characterised by the presence of bilateral small kidneys with numerous small cortico-medullary cysts which give an appearance of increased parenchymal echogenicity, with loss of the corticomedullary junction. Although rare, ADPKD can present in this age group.

How well did you know this?
1
Not at all
2
3
4
5
Perfectly
65
Q

63 A 50 year old woman presents with vague left sided· abdominal pain. KUB demonstrates no renal tract calcificationbut an enlarged left renal outline is noted. Which of the following is not part of the differential diagnosis?

(a) Cervical carcinoma

(b) Nephrotic syndrome

(c) Pyelonephritis

(d) Congenital duplication

(e) Renal artery dissection

A

(e) Renal artery dissection

Renal artery dissection causes the affected kidney to decrease in size due to arterial insufficiency. Ureteric obstruction with hydronephrosis can be a presenting feature of cervical cancer.

How well did you know this?
1
Not at all
2
3
4
5
Perfectly
66
Q

65 A 25 year old woman presents with a 6 month history of debilitating right loin pain. IVU and arteriography show a vascular impression on the superior infundibulum with secondary dilatation of the upper pole calyx. Which of the following is the likeliest diagnosis?

(a) Fraley’s syndrome

(b) Cavernous haemangioma

(c) Bartter syndrome

(d) Bardet-Biedel syndrome

(e) Alport syndrome

A

(a) Fraley’s syndrome

Fraley’s syndrome is rare, comprising superior infundibular narrowing due to a crossing vessel causing significant loin pain. It is more common in young women and on the right side. If symptomatic surgery is indicated.

How well did you know this?
1
Not at all
2
3
4
5
Perfectly
67
Q

69 A 30 year old man is involved in a high speed RTA. At contrast enhanced CT, he is noted to have a laceration in the upper pole of his right kidney extending through the corticomedullary junction into the collecting system with extravasation and surrounding haematoma. According to the American Association for the Surgery of Trauma guidelines, how would you categorise this injury?

(a) Grade I

(b) Grade II

(c) Grade Ill

(d) Grade IV

(e) Grade V

A

(b) Grade II

Grade I refers to contusion or non-expanding subcapsular haematoma and is the commonest type.
Grade II refers a laceration less than 1cm deep without extravasation.
Grade III describes a laceration more than 1 cm deep without extravasation.
Grade IV refers to the above or alternatively to a segmental renal artery or vein injury with contained haematoma.
Grade V refers to a shattered kidney or to renal pedicle injury

How well did you know this?
1
Not at all
2
3
4
5
Perfectly
68
Q

75 A 35 year old man undergoes renal transplantation. On the 1st post operative day he becomes oliguric. Doppler ultrasound shows an enlarged, generally hypoechoic graft with diffusely diminished cortical perfusion, decreased systolic ri~e time and a plateau like reversal of diastolic flow. Which of the following is the likeliest diagnosis?

(a) Normal day 1 post transplant changes

(b) Renal allograft necrosis

(c) Renal vein thrombosis

(d) Renal artery stenosis

(e) Arteriovenous fistula

A

(c) Renal vein thrombosis

Renal/iliac vein thrombosis occurs in up to 5% of cases. If it occurs in the immediate post-operative period it is usually due to endothelial injury at the anastomosis site or extrinsic compression. An absence of venous flow can also be observed with Doppler US.

How well did you know this?
1
Not at all
2
3
4
5
Perfectly
69
Q
  1. A 23 year old female has a renal ultrasound scan for recurrent urinary tract infections. The only abnormality detected is a 3 cm hyperechoic mass in the upper pole of the left kidney. She subsequently undergoes CT which shows the lesion to have an average HU of –10. Which of the following is the most likely diagnosis?

a. Renal cell carcinoma

b. Transitional cell carcinoma

c. Renal lymphoma

d. Angiomyolipoma

e. Renal abscess

A
  1. d. Angiomyolipoma

The finding of fat attenuation values within a renal lesion on CT is diagnostic of angiomyolipoma. This is a benign tumour that is typically hyperechoic on ultrasound and of high signal on T1-weighted MR due to fat. It does not enhance post-gadolinium, in contrast to renal cell carcinoma, which usually does enhance

How well did you know this?
1
Not at all
2
3
4
5
Perfectly
70
Q
  1. A 65 year old male has a renal ultrasound scan for right flank pain which demonstrates a 7 cm solid mass within the right kidney with a hypoechoic centre. Subsequent CT scan of the chest, abdomen and pelvis reveals the lesion to have a low-attenuation central scar. There is no renal vein invasion or evidence of malignancy elsewhere in the body. Which of the following is the most likely diagnosis?

a. Lymphoma of the kidney

b. Transitional cell carcinoma

c. Collecting duct tumour

d. Oncocytoma

e. Nephroblastoma

A
  1. d. Oncocytoma

The features described are typical of renal oncocytoma. Oncocytoma is a tubular adenoma that is very rarely malignant. They are often asymptomatic even when large. The central scar is typical and is due to haemorrhage and infarction of the tumour having outgrown its vascular supply. Radiological differentiation from renal cell carcinoma can be very difficult and percutaneous needle biopsy is unreliable. Nephrectomy is therefore often indicated

How well did you know this?
1
Not at all
2
3
4
5
Perfectly
71
Q
  1. A 45 year old male is diagnosed with renal cell carcinoma and is being worked up for curative nephrectomy. Which one of the following imaging modalities would you advise as being the most accurate at ruling out malignant renal vein invasion?

a. Doppler ultrasound

b. B-mode ultrasound

c. CT

d. MRI

e. PET-CT

A
  1. d. MRI

MRI is superior to the other imaging modalities listed at ruling out renal vein invasion. CT is still very accurate (reported as high as 96%), but MR has the advantage of being able to accurately differentiate benign from malignant thrombus. MR offers no advantage in detecting nodal disease, however, and patients being considered for curative surgery should undergo staging CT of the chest, abdomen and pelvis. PET does not have a specific role for detecting renal vein invasion

How well did you know this?
1
Not at all
2
3
4
5
Perfectly
72
Q
  1. A 71 year old male undergoes renal CT for characterisation of a cystic renal mass. Which one of the following five features would classify the lesion as a Bosniak III lesion?

a. Lack of enhancement

b. Septation

c. Minimally irregular wall

d. Curvilinear calcification

e. Uniform wall thickening

A
  1. e. Uniform wall thickening

The Bosniak classification groups cystic renal lesions into one of four categories based on CT/MR appearances. The differentiation between groups II and III is important as group II are typically ‘follow-up lesions’ and group III are ‘surgical lesions’. Features of a Bosniak III lesion include irregular thickened septa, measurable enhancement, coarse irregular calcification, multiloculation, nodularity, uniform wall thickening and margin irregularity

How well did you know this?
1
Not at all
2
3
4
5
Perfectly
73
Q
  1. A 56 year old female presents with a three-month history of pyrexia, loin pain and weight loss. Urinalysis reveals pyuria and haematuria. Urinary culture reveals Proteus mirabilis. A renal CT demonstrates a globally enlarged kidney with extensive perirenal inflammation, an absent nephrogram and a staghorn calculus. Which one of the following diagnoses is most likely?

a. Leukoplakia

b. Emphysematous pyelonephritis

c. Pyeloureteritis cystica

d. Xanthogranulomatous pyelonephritis

e. Page kidney

A
  1. d. Xanthogranulomatous pyelonephritis

The most likely diagnosis is xanthogranulomatous pyelonephritis. This is a chronic granulomatous infection in a chronically obstructed kidney, often secondary to a staghorn calculus. It often presents insidiously in middle-aged to elderly females. It is most commonly diffuse, but the focal form seen in 15% of cases may provide a diagnostic dilemma as it can be difficult to confidently distinguish from renal cell carcinoma

74
Q
  1. A 32 year old male presents with right flank pain and an intravenous urogram is requested with the provisional diagnosis of ureteric calculi. The renal outline is smooth and wavy, with a decreased overall size. The fornices are widened with club-shaped calyces. After further questioning he reveals a recent overuse of analgesia. Which one of the following diagnoses is most likely?

a. Acute cortical necrosis

b. Acute tubular necrosis

c. Papillary necrosis

d. Acute interstitial nephritis

e. Haemorrhagic cystiti

A
  1. c. Papillary necrosis

Papillary necrosis occurs due to ischaemic damage to the medulla of the kidney, and does not primarily involve the cortex. There are many causes including diabetes, analgesic nephropathy, pyelonephritis, renal vein thrombosis and sickle cell disease. It may be localised or diffuse, bilateral or unilateral depending on the cause. Intravenous urogram appearances are varied and include clubbed calyces, calcification and sloughing of necrotic papilla and alteration in the renal contour.

75
Q
  1. A 21 year old female undergoes a renal ultrasound scan following an abdominal X-ray that demonstrated multiple foci of calcification in both renal areas. The ultrasound reveals multiple medullary cysts bilaterally which are seen to communicate with the collecting system. The medullae of both kidneys are of increased echogenicity. Which of the following diagnoses is most likely?

a. Megacalicosis

b. Multicystic dysplastic kidney

c. Autosomal dominant polycystic kidney disease

d. Autosomal recessive polycystic kidney disease

e. Medullary sponge kidney

A
  1. e. Medullary sponge kidney

The features are those of medullary sponge kidney. This is a non-inheritable condition that produces cystic dilatation of the collecting ducts and nephrocalcinosis; 75% of cases are bilateral and it is usually asymptomatic. It is associated with an increased incidence of infection and urolithiasis but is not thought to predispose to malignancy

76
Q
  1. A 65 year old male undergoes renal CT following the finding of multiple hypoechoic masses in both kidneys on ultrasound. Multiple poorly defined masses of decreased attenuation are demonstrated, which encase the renal vessels. The vessels remain patent, however, and the renal contour is preserved. Which of the following is most likely to represent the underlying diagnosis?

a. Renal cell carcinoma

b. Transitional cell carcinoma

c. Multiple myeloma

d. Non-Hodgkin’s lymphoma

e. Reninoma

A
  1. d. Non-Hodgkin’s lymphoma

The most likely diagnosis is non-Hodgkin’s lymphoma. Primary renal involvement is rare, but is often involved either by haematogenous spread or direct invasion. The kidneys represent one of the most common extra-nodal sites of disease in non-Hodgkin’s lymphoma, but are rarely involved in Hodgkin’s disease. Involvement is usually bilateral, with masses of decreased attenuation and mild homogenous enhancement with intravenous contrast on CT. Patency of the renal vessels despite encasement is highly suggestive, as is preservation of the normal renal contour

77
Q
  1. A 43 year old female has a renal ultrasound which shows a left-sided renal ‘mass’. The ‘mass’ is continuous with the renal cortex and has the same echogenicity as the cortex. It is situated at the border of the upper and mid poles of the left kidney and is seen to extend between the renal pyramids. Which one of the following are these features most likely to represent?

a. Renal scarring

b. Hypertrophied column of Bertin

c. Dromedary hump

d. Persistent fetal lobulation

e. Duplex kidney

A
  1. b. Hypertrophied column of Bertin

Many lesions may be mistaken for a renal cell carcinoma on imaging, and it is important to be able to differentiate such ‘pseudotumours’ from genuine carcinomas. The features described in the question are consistent with a prominent column of Bertin. This is normal renal tissue located between the pyramids and extending into the renal sinus. The key features include continuity with the cortex, identical echogenicity to normal cortex and the lack of mass effect or renal outline deformity. A dromedary hump is a focal bulge on the lateral border of the left kidney caused by its relationship with the adjacent spleen. Persistent fetal lobulation can be identified by indentations of the renal surface that overlie the space between the pyramids, whereas renal scarring lies directly over the medullary pyramids

78
Q
  1. A 31 year old female is admitted to hospital with placental abruption. Her renal function deteriorates significantly and therefore a renal ultrasound is requested. Kidneys with bilateral increased echogenicity and thin tramline calcification of the cortices are seen. Which of the following underlying conditions is most likely?

a. Acute cortical necrosis

b. Papillary necrosis

c. Barter syndrome

d. Drug-related nephrotoxicity

e. Renal infarction

A
  1. a. Acute cortical necrosis

The features described are typical of cortical nephrocalcinosis. Causes of cortical nephrocalcinosis include acute cortical necrosis, chronic glomerulonephritis, sickle cell disease, Alport syndrome and congenital oxalosis. Acute cortical necrosis is a rare cause of acute renal failure, and is most commonly due to complications of pregnancy such as placental abruption, infected abortion and severe eclampsia

79
Q

QUESTION 7
A 32-year-old man received a cadaveric renal transplant 3 days ago. He now presents with increasing right iliac fossa pain and deteriorating renal function. On ultrasound, there is mild dilatation of the pelvicalyceal system with prominent renal pyramids. On Doppler ultrasound, colour Doppler flow is present within the renal artery and the interlobar arteries. The interlobar arterial waveform has tall systolic peaks with diastolic flow below the baseline. What is the most likely diagnosis?

A Acute tubular necrosis

B Acute rejection

C Ar teriovenous fistula

D Renal artery stenosis at the site of anastomosis

E Renal vein thrombosis

A

E Renal vein thrombosis

Diastolic flow reversal in the renal arcuate arteries on spectral Doppler ultrasound is a feature of renal vein thrombosis. Acute tubular necrosis (ATN) and acute rejection are difficult to distinguish on a single Doppler examination because they both cause an increase in the resistive index (> 0.8). Renal artery stenosis does not cause diastolic flow reversal and classically produces a slow rising parvus et tardus waveform. An arteriovenous fistula is usually the result of renal biopsies and presents as an area of turbulent high flow within the renal parenchyma

80
Q

QUESTION 9
A 40-year-old male diabetic patient has an intravenous urogram (IVU) for left sided renal colic. On the IVU, the left kidney shows papillary and calyceal abnormalities that give an ‘egg in a cup’ appearance at some calyces and ‘tracks and horns’ at other calyces. The affected left kidney has preserved renal cortical thickness despite the calyceal/papillary abnormalities. The contralateral kidney appears normal. What is the most likely diagnosis?

A Acute pyelonephritis

B Amyloidosis

C Reflux nephropathy

D Renal papillary necrosis

E Xanthogranulomatous pyelonephritis

A

D Renal papillary necrosis

81
Q

QUESTION 10
A 40-year-old female diabetic patient has right loin pain, vomiting and a fever. An ultrasound examination is requested to exclude urinary obstruction. This demonstrates no evidence of upper tract dilatation, but features of acute pyelonephritis are present. What are the most likely sonographic findings within the right kidney?

A Focal areas of reduced reflectivity in the renal parenchyma

B Focal atrophy of segments of the right kidney

C Increased echogenicity of the renal calyces

D Enlarged right kidney and diffusely hyperechoic parenchyma

E Shrunken right kidney and diffusely hyperechoic parenchyma

A

A Focal areas of reduced reflectivity in the renal parenchyma

82
Q

QUESTION 11
A 43-year-old female diabetic patient with right-sided renal colic has a CT urogram (CTU). This demonstrates a calculus within the right renal pelvis and the right renal cortex is almost entirely replaced by a heterogeneous illdefined mass that extends to involve Gerota’s fascia. This right renal mass contains rounded areas of low attenuation (-15 to -20 HU) which don’t enhance postcontrast. On a subsequent MRI, the right renal mass appears heterogeneous with rounded areas of high signal on Tlw and low signal on STIR images. What is the most likely diagnosis?

A Emphysematous pyelonephritis

B Renal angiomyolipoma

C Renal cell carcinoma

D Staghorn calculus with coexistent malakoplakia

E Xanthogranulomatous pyelonephritis.

A

E Xanthogranulomatous pyelonephritis.

Xanthogranulomatous pyelonephritis (XGP) is a rare form of low-grade chronic renal infection, characterised by progressive destruction of the renal parenchyma. The histological hallmark is replacement of the renal parenchyma by lipid-laden foamy histiocytes. Women are predominantly affected (in a 3:1 ratio), usually in mid-life (50—60 years) and after a long history of recurrent urinary tract infection or urinary stones. Involvement of both kidneys is exceedingly rare. XGP is diffuse in 90% of cases, but focal XGP can simulate a renal tumour.

83
Q

QUESTION 13
A 55-year-old HIV-positive man presents with macroscopic haematuria and right-sided renal colic. An IVU does not demonstrate any renal tract calcification, but there is a dense right nephrogram with no excretion of contrast on a delayed film. The urologist performs a retrograde ureteroscopy and retrieves a 9-mm right ureteric calculus. What is the likely composition of the calculus?

A Calcium oxalate

B Cysteine

C Indinavir phosphate

D Struvite

E Uric acid

A

C Indinavir phosphate

84
Q

QUESTION 16
A patient with normal renal function and suspected right renal artery stenosis undergoes a dynamic angiotensin converting enzyme inhibitor (ACE1) MAG3 renogram. Which one of the following statements best describes the findings of the ACEI renogram in right renal artery stenosis?

A After administration of the ACEI, there is increased blood flow to the right kidney.

B On a time activity graph, the mean transit time of the right kidney is reduced following ACEI.

C On a time activity graph, the time to peak of the right kidney is increased following ACEI.

D The ACEI renogram will be normal because the renal function is preserved.

E The ACEI renogram will show an increase in total and relative renal function of the right kidney when compared with the left kidney.

A

C On a time activity graph, the time to peak of the right kidney is increased following ACEI.

85
Q

QUESTION 17
A 27-year-old man with membranous glomerulonephritis presents with a 1-day history of right-sided flank pain and haematuria. An abdominal radiograph did not reveal any renal calcification but his renal function has significantly deteriorated over the past 24 hours. On ultrasound there is a large, oedematous right kidney with loss of the corticomedullary differentiation. On a subsequent IVU, there is a faint nephrogram with absent pelvicalyceal filling after 15 minutes. What is the most likely diagnosis?

A Acute hydronephrosis

B Acute pyelonephritis

C Acute renal infarction

D Acute renal vein thrombosis

E Chronic pyelonephritis

A

D Acute renal vein thrombosis

86
Q

QUESTION 19
A 57-year-old diabetic man with a 6-week history of pyrexia of unknown origin has a contrast-enhanced CT abdomen. There is an upper pole renal mass that has a thick irregular enhancing wall with a central area of fluid attenuation with other areas of very low attenuation (—1000 HU). Which one of the following is the likely diagnosis?

A Metastatic disease

B Renal abscess

C Renal cyst with haemorrhage

D Renal infarction

E Renal lymphoma

A

B Renal abscess

The presence of gas within a lesion is virtually diagnostic of an abscess but very rarely seen. Immunocompromised patients and those with diabetes are more prone to renal abscess.

87
Q

QUESTION 20
A 74-year-old woman undergoes a follow-up CT for gastric carcinoma. She has previously received palliative chemotherapy, but has recently deteriorated with marked loss of weight. The CT demonstrates progressive disease with new lung and liver metastases and there are new abnormalities in both kidneys. Which one of the following statements best describes the CT findings of haematogenous metastases to the kidneys?

A Curvilinear (arc)-like calcification is a characteristic feature.

B Metastases to the kidney are usually < 3 cm in size and limited to the cortex

C Multiple lesions involving the medulla are a feature of haematogenous metastases.

D If renal vein invasion is not present, renal metastases are highly unlikely.

E Renal metastases are usually hypovascular on contrast-enhanced CT.

A

B Metastases to the kidney are usually < 3 cm in size and limited to the cortex

88
Q

(Ped) QUESTION 22
A 5-year-old child presents with pyrexia and weight loss. The urine cultures reveal Proteus mirabilis. An ultrasound demonstrates an enlarged kidney, consistent with an inflammatory mass. Xanthogranulomatous pyelonephritis is considered. Which one of the following statements is true regarding the imaging findings of this condition?

A Calcification is rarely seen on CT.

B Hot spots are seen on a Tc-99m DMSA scan.

C On MRI the necrotic areas tend to be high signal on T2w images with medium signal intensity on Tlw images.

D Perinephric extension is unusual on both CT and MRI.

E The infected cavities enhance avidly with intravenous contrast on CT.

A

C On MRI the necrotic areas tend to be high signal on T2w images with medium signal intensity on Tlw images.

It is common to see calcification within the inflammatory mass as well as associated ureteric calcification. The abscess cavities are low attenuation on CT, with surrounding rim enhancement

89
Q

QUESTION 23
An 81-year-old woman presents to the Emergency Department with sepsis and left iliac fossa pain. She has grossly elevated inflammatory markers and serum creatinine = 212 ml mol/L. A contrast-enhanced CT is requested for suspected acute diverticulitis, but the on-call radiology SpR is concerned about the possibility of contrast-mediated nephrotoxicity (CMN). Which one of the following statements is true regarding CMN?

A Atrial fibrillation is an independent risk factor for developing CMN.

B CMN is defined as renal impairment (an increase in serum creatinine by more than 50% or by 44 pmol/L above baseline) within 24 hours of contrast injection.

C Low osmolar contrast media is more nephrotoxic than high osmolar contrast media, in patients with pre-existing renal impairment.

D Pre-hydration with 1 mL/kg body weight/h of 0.9% NaCl for 4 hours prior to the contrast injection may increase the incidence of CMN.

E Prophylactic haemodialysis in patients with renal impairment does not reduce the risk of CMN.

A

E Prophylactic haemodialysis in patients with renal impairment does not reduce the risk of CMN.

CMN is defined as an increase in serum creatinine by more than 25% or by 44[lmol/L above baseline within 3 days of contrast administration. Prophylactic haemodialysis does not reduce the incidence of CMN but haemofiltration does.

90
Q

@#e QUESTION 29
A 35-year-old patient received a cadaveric renal transplant 5 days ago and now presents with worsening renal function and decreasing urine output. Which one of the following findings on a Tc-99m DTPA radionuclide scan would favour a diagnosis of acute tubular necrosis (ATN) over acute rejection?

A Delayed renal excretion

B Elevated resistive index greater than 0.7

C Increased renal perfusion after administration of an ACEI (eg Captopril)

D Poor/impaired graft perfusion

E Preserved renal transplant perfusion

A

E Preserved renal transplant perfusion

ATN is an early complication in cadaveric allografts and frequently resolves spontaneously in 1—3 weeks. The radionuclide imaging findings of ATN are of preserved perfusion but poor renal function and urine excretion. In acute rejection however, there is both impaired renal function and reduced perfusion on radionuclide imaging.

91
Q

QUESTION 31
A 34-year-old man is knocked off his bicycle by a car and presents to the Emergency Department with bruising over the right flank and gross haematuria. The A&E SpR suspects renal injury and requests a CT abdomen. Which one of the following findings is most likely to be seen in uncomplicated renal contusion (Grade 1 renal injury)?

A Ill-defined areas of low attenuation with irregular margins

B Subcapsular high attenuation collection

C Wedge-shaped areas of high attenuation, typically involving the renal

D Well-defined areas of low attenuation within the renal parenchyma

E Urinoma formation

A

A Ill-defined areas of low attenuation with irregular margins

92
Q

QUESTION 32
A 24-year-old motorcyclist involved in a traffic accident presents to the Emergency Department with a broken leg and bruising over his left flank. He is found to have microscopic haematuria and fractures of the left 8th and 9th ribs. The patient is haemodynamically stable and clinicians suspect a left renal injury. Which one of the following imaging investigations is the most appropriate?

A Abdominal ultrasound

B Contrast-enhanced CT abdomen and pelvis

C Emergency catheter renal angiography

D Gadolinium-enhanced renal MRI

E IVU

A

B Contrast-enhanced CT abdomen and pelvis

In the setting of a haemodynamically stable patient with a history of major blunt trauma, CT is the most appropriate imaging investigation

93
Q

@#e QUESTION 35
A 24-year-old motorcyclist is involved in a high-speed accident and is brought to the Emergency Department. He has abdominal guarding and is haemodynamically unstable. An ultrasound abdomen performed in the Emergency Department demonstrates free peritoneal fluid and a laparotomy is performed. In addition to liver and splenic lacerations, the surgeon finds a left retroperitoneal haematoma. Postoperatively, the on-call urologist requests a CT abdomen to assess the left renal injury. Which one of the following findings would indicate a Grade 4 renal laceration?

A Extravasation of contrast from the pelvicalyceal system on delayed phase (5 min) images

B Large (2-cm) subcapsular haematoma

C Perinephric haematoma that extends into the pararenal spaces

D Ill-defined low attenuation change in the lower pole renal cortex

E Segmental renal infarction

A

A Extravasation of contrast from the pelvicalyceal system on delayed phase (5 min) images

A deep renal laceration that extends into the collecting system is indicative of a grade 4 injury

94
Q

QUESTION 37 Which one of the following statements best describes the radiological appearances of a parapelvic renal cyst?

A It does not opacify during IVU.

B If hydronephrosis is present, a parapelvic cyst can be excluded.

C It shows delayed (10 min) filling on IVU.

D It may have similar appearances to calyceal diverticula on IVU.

E The majority arise from the lower renal pole.

A

A It does not opacify during IVU.

A parapelvic cyst is located near the renal hilum, does not communicate with the renal pelvis (unlike calyceal diverticula) and therefore does not opacify during IVU. It compresses the pelvis and may cause hydronephrosis.

95
Q

@#e QUESTION 38
A 64-year-old man has an abdominal MRI to further characterise a well-defined 2.5-cm solid renal mass at the left upper pole. The lesion was hyperechoic on ultrasound. Which one of the following MRI findings would favour a diagnosis of angiomyolipoma?

A High signal on Tlw and low signal on T2w sequences

B High signal on Tlw and STIR sequences

C High signal on Tlw and T2w sequences

D High signal on T2w and low signal on proton density sequences

E Low signal on both T2w and proton density sequences

A

C High signal on Tlw and T2w sequences

Angiomyolipomas appear high signal on both Tlw and T2w sequences due to their high fat content

96
Q

QUESTION 39
A 42-year-old man is referred for investigation of painless microscopic haematuria. An IVU is performed and demonstrates bilateral small areas of calcification within the kidneys on the control image. On the 5-min postcontrast IVU film, the calcification appears to lie within the collecting system. On ultrasound, there are numerous small hyperechoic rounded areas within the medullary pyramids, many of which cast an acoustic shadow. What is the most likely diagnosis?

A Adult polycystic kidney disease

B Hyperparathyroidism

C Medullary sponge kidney

D Primary hyperoxaluria

E Sarcoidosi

A

C Medullary sponge kidney

In medullary sponge kidney the kidneys contain numerous medullary cysts which communicate with the tubules and therefore opacify during excretion urography. The cysts contain small calculi giving a ‘bunch of grapes appearance

97
Q

QUESTION 43
Which one of the following statements best describes the CT appearances of a renal oncocytoma (tubular adenoma)?

A It appears as a small, ill-defined renal mass in the majority of cases.

B It is bilateral in 60-80% of cases.

C It characteristically consists of multiple renal lesions.

D CT shows punctuate calcification in the majority of patients.

E Low attenuation (—100 to —50 HU) areas within a large lesion are consistent with an oncocytoma.

A

E Low attenuation (—100 to —50 HU) areas within a large lesion are consistent with an oncocytoma.

Large lesions can extend into and engulf the perinephric fat, and can therefore be mistaken for angiomyolipomas (due to fat content).

98
Q

QUESTION 47
A 54-year-old man has an abdominal ultrasound that shows a 3-cm hyperechoic lesion at the upper pole of the left kidney. An unenhanced CT abdomen is subsequently performed and demonstrates a left upper pole heterogeneous renal mass with central areas of low attenuation (5—10 HU). After intravenous contrast is administered, this mass enhances by more than 30 HU. What is the most likely diagnosis?

A Angiomyolipoma

B Oncocytoma

C Renal abscess

D Renal cell carcinoma

E Unilocular renal cyst

A

D Renal cell carcinoma

Angiomyolipomas are benign, fat-containing lesions which do not enhance by more than 15 HU and contain low attenuation (—15 to —20HU) fatty areas. Postcontrast enhancement of greater than 20 HU of a solid renal mass is highly suggestive of malignancy.

99
Q

QUESTION 50
A 42-year-old man with known Wegener’s granulomatosis develops haematuria. He has an abdominal ultrasound which reveals small, smooth kidneys with diffuse thinning of the renal parenchyma. The pelvicalyceal systems appear normal but there is an increased amount of renal sinus fat. What is the most likely diagnosis?

A Bilateral vesicoureteric reflux

B Chronic glomerulonephritis

C Medullary sponge kidney

D Pyelonephritis

E Renal tuberculosis

A

B Chronic glomerulonephritis

100
Q

@#e QUESTION 51
A 22-year-old pregnant woman (30 weeks’ gestation) presents with right flank pain. She has an abdominal ultrasound which shows dilatation of the right pelvicalyceai system. Which one of the following additional findings would suggest a diagnosis of mechanical ureteric obstruction rather than pregnancy-related dilatation?

A An elevated resistive index (RI)

B Decreased corticomedullary differentiation

C Hyperechoic renal parenchyma

D Renal parenchymal thinning

E Ureteric and pelvicalyceai dilatation

A

A An elevated resistive index (RI)

Mechanical obstruction is associated with elevation of the RI.

101
Q

QUESTION 53
A 29-year-old man presents with a 4-hour history of sudden onset right loin pain, radiating to the right groin. The clinicians request an emergency IVU for suspected acute urinary obstruction. Which one of the following IVU features would be most consistent with acute urinary obstruction?

A Absent right nephrogram and no evidence of contrast excretion on the right

B An increasingly dense right nephrogram that remains present after 6 hours

C An initially dense right nephrogram, which then resolves within 30 minutes

D The right kidney being 10% longer than the left kidney

E The right kidney being small with an irregular cortical surface

A

B An increasingly dense right nephrogram that remains present after 6 hours

102
Q

(Ped) QUESTION 56
A 2-week-old septic neonate shows worsening renal function and proteinuria. He is currently being monitored on the paediatric ITU. Seven days after his initial illness, an ultrasound is performed which reveals a unilateral enlarged kidney, with loss of corticomedullary differentiation and reversal of end diastolic arterial flow. Associated adrenal haemorrhage is noted. What is the most likely diagnosis?

A Acute glomerulonephritis

B Acute tubular necrosis

C Renal artery stenosis

D Renal vein thrombosis

E Unilateral obstruction

A

D Renal vein thrombosis

The findings are all consistent with renal vein thrombosis, which can be seen in the dehydrated septic neonate. Ultrasound changes of acute tubular necrosis are usually symmetrical.

103
Q

QUESTION 70
A 33-year-old man presents with left renal colic. An abdominal radiograph is normal but a subsequent CT urogram demonstrates an obstructing, opaque 10-mm distal left ureteric calculus. What is the renal calculus most likely to be composed of?

A Calcium oxalate

B Calcium phosphate

C Cysteine

D Pure matrix

E Uric acid

A

E Uric acid

Uric acid stones are not visible on plain radiographs but are seen on CT.

104
Q
  1. E. coli, with or without an underlying diagnosis of diabetes mellitus, is the most common pathogen behind many urological conditions. All of the following conditions are most commonly due to E. coli infection except one, which one?

A. Xanthogranulomatous pyelonephritis (XGP).

B. Emphysematous pyelonephritis.

C. Pyonephrosis.

D. Pyeloureteritis cystica.

E. Malakoplakia.

A
  1. A. Xanthogranulomatous pyelonephritis (XGP).

This is most commonly secondary to proteus infection. XGP is discussed elsewhere in this chapter.

105
Q
  1. A 64-year-old female patient presents with a 3-month history of right-sided loin pain, microscopic haematuria, and low-grade pyrexia. Her GP had been treating the patient as a presumptive UTI, given the background history of diabetes, but is concerned that the symptoms are not resolving. On clinical examination the urologists have felt a right-sided ballotable loin mass. They have requested a CT scan of abdomen. This reveals a large irregular mass replacing most of the parenchyma of the right kidney, which measures 10 × 11 cm in axial diameter and does not demonstrate significant enhancement. There are a number of focal masses noted within the dominant mass that have attenuation values of between –15 and –5 HU. The renal outline is poorly defined, with involvement of the perirenal fascia. A large staghorn calculus is also present. There is no evidence of invasion of the renal vein. What is the primary diagnosis?

A. Renal cell carcinoma.

B. Renal oncocytoma.

C. Renal lymphoma.

D. XGP.

E. Renal angiomyolipoma.

A
  1. D. XGP.

Whilst the patient presents with the classical renal cell carcinoma (RCC) triad of loin pain, haematuria, and a mass, this triad is actually only present in 30% of patients with RCC. These features are also typically found in XGP. The key finding in this case is of areas of fatty attenuation within the mass, in association with a staghorn calculus. Calculi are found in 80–90% of cases of XGP. The long history of symptoms is also typical of XGP, which can be present for up to 6 months prior to diagnosis. Lymphoma, RCC, and renal oncocytoma would not typically have areas of fatty tissue within them. Angiomyolipomas, in the absence of tuberous sclerosis, are typically solitary. They are usually well defined, unlike in this case, and would rarely reach this size without having previously caused symptoms. XGP is typically found in diabetic patients and is due to proteus infection. Treatment is with nephrectomy.

106
Q
  1. A 67-year-old male patient presents with an 8-week history of left loin pain. A renal CT is obtained and this shows a 6-cm enhancing left renal lesion that has a fibrotic central scar. What is the most likely diagnosis?

A. Renal leiomyoma.

B. Renal oncocytoma.

C. Renal metanephic adenoma.

D. Renal haemangioma (giant).

E. Renal juxta-glomerular cell neoplasm

A
  1. B. Renal oncocytoma.

This is a benign renal cell neoplasm responsible for about 5% of all adult primary renal epithelial neoplasms. It typically occurs in elderly men. They usually appear as solitary, well-demarcated, unencapsulated, fairly homogeneous renal cortical tumours. Bilateral, multicentric oncocytomas are seen in hereditary syndromes of renal oncocytosis and Birt–Hogg–Dubé syndrome. A central stellate scar is seen in approximately one-third. However, distinguishing them from RCC on imaging is not reliable. Leiomyoma of the kidney is a benign smooth muscle neoplasm. It appears as a well-circumscribed, homogeneous, exophytic solid mass that shows uniform enhancement on contrast-enhanced CT. It may occasionally be cystic. Metanephric adenoma is a benign renal neoplasm that is more common in middle-aged to elderly females. It is associated with polycythaemia in 10%. It typically appears as a well-defined, unencapsulated, solitary mass that may be hyperattenuating on unenhanced CT. Calcification can be seen in up to 20%. Hemangioma of the kidney occurs as an unencapsulated, solitary lesion that frequently arises from the renal pyramids or the pelvis. Contrast-enhanced CT or MRI may show early intense enhancement, with persistent enhancement on delayed images. Juxtaglomerular cell (JGC) neoplasm or reninoma is an extremely rare, benign renal neoplasm of myoendocrine cell origin, which is associated with a clinical triad of hypertension, hypokalaemia, and high plasma renin activity. It typically appears as a unilateral, well-circumscribed, cortical tumour and often measures less than 3 cm, but otherwise is indistinguishable from other cortical neoplasms.

107
Q

32
15. A 54-year-old female patient with diabetes presents with a history of left loin pain, pyrexia, and pyuria over the past 24 hours. The patient develops sepsis soon after admission and is brought for a CT scan following appropriate resuscitation. The CT shows a mildly dilated left renal pelvis, which contains small pockets of air extending into the calyces. A small wedge-shaped area of poor enhancement in the lower pole is the only parenchymal abnormality. The clinical diagnosis is of pyelonephritis, but what subtype of pyelonephritis is best represented by these imaging features?

A. Acute pyelonephritis.

B. Emphysematous pyelitis.

C. Emphysematous pyelonephritis.

D. Acute pyonephrosis.

E. XGP.

A
  1. B. Emphysematous pyelitis.

The combination of these radiological findings in a septic diabetic patient indicates an infection with a gas-forming organism, usually E. coli. The gas is located entirely within the pelvicalyceal system, as opposed to the renal parenchyma, which would indicate a diagnosis of emphysematous pyelonephritis. The importance is in the urgency of treatment and prognosis. Emphysematous pyelonephritis carries a grave prognosis and often requires nephrectomy for life-saving treatment, whereas the prognosis with emphysematous pyelitis is not as grave and it often responds to antibiotics with possible urinary drainage procedures if necessary

108
Q
  1. A 40-year-old man has an ultrasound examination of the abdomen for epigastric pain. The examination is normal apart for an isoechoic ‘mass’ at the midpole of the left kidney. You suspect this may be a normal variant, such as a prominent column of Bertin. You recommend a CT scan of kidneys to confirm this. Which phase of a CT renal tract examination would best show this variant?

A. Unenhanced scan.

B. Cortico-medullary phase.

C. Nephrographic phase.

D. Early excretory (3-minute) phase.

E. Late excretory (10-minute) phase

A
  1. B. Cortico-medullary phase.

Imaging in the cortico-medullary phase is helpful for showing the normal cortico-medullary pattern in pseudotumours such as prominent columns of Bertin or focal renal hypertrophy. It is also useful for suspected abnormalities such as vascular malformations and pseudoaneurysms of the kidney. The nephrographic phase is best to characterize renal masses. Small parenchymal renal lesions may be hidden in the renal medulla on the cortico-medullary phase and excretory phases. Excretory phases are best to assess for collecting system lesions and the unenhanced scan is best for detection of renal calculi and for using as a baseline in assessing the enhancement characteristics of parenchymal renal lesions.

109
Q
  1. You are consulted about a 24-year-old male patient who is day 3 following a blind right-sided renal biopsy, which diagnosed glomerulonephritis. He has been well until today, when he developed acute right flank pain and haematuria. The urologists asked for an IVU prior to assessing the patient. They feel this showed an absent kidney, but on the 15- and 20-minute images, you feel you can detect a faint nephrogram that is slightly more pronounced on the 20-minute image. Given your suspicions about the cause, which of the following would be the least likely to contribute to the diagnosis?

A. Ultrasound.

B. MRI with GE sequences of the right kidney.

C. MRI with SE sequences of the right kidney.

D. Arterial phase CT abdomen.

E. Portal venous phase CT abdomen.

A
  1. D. Arterial phase CT abdomen.

The IVU has surprisingly been helpful in showing a delayed increasing nephrogram, a classical, although uncommon, appearance of renal vein thrombosis. This is also indicated by the history of glomerulonephritis, which can manifest as nephrotic syndrome, the most common cause for renal vein thrombosis. Ultrasound is often successful at identifying a thrombosed right renal vein, but is not useful in assessing the longer left renal vein. Gradient and SE sequences are both helpful in assessing the renal venous system, with GE giving low-signal thrombus and SE giving high-signal thrombus. Arterial phase CT would be too early to show a venous filling defect and thus may not contribute, although it may show a rim nephrogram. Portal venous phase CT is reasonably sensitive at detecting the lack of enhancement within the renal vein and abnormal enhancement pattern within the affected kidney

110
Q
  1. A 40-year-old male with a history of haematuria undergoes CT urography. Initial non-contrast scan demonstrates right-sided medullary nephrocalcinosis. Following intravenous contrast administration, a striated ‘paintbrush’ appearance of the renal medulla is noted. The left kidney is unremarkable. What is the diagnosis?

A. Hyperparathyroidism.

B. Renal tubular acidosis.

C. Medullary sponge kidney.

D. Sarcoidosis.

E. Multiple myeloma.

A
  1. C. Medullary sponge kidney.

Hyperparathyroidism, renal tubular acidosis, and medullary sponge kidney are the three most common causes of medullary nephrocalcinosis. The former two conditions are associated with hypercalciuria that results in uniform medullary nephrocalcinosis. Sarcoidosis and multiple myeloma are associated with hypercalcemia resulting in bilateral nephrocalcinosis. Medullary sponge kidney can affect the kidney segmentally, unilaterally, or bilaterally, therefore unilateral nephocalcinosisis is suggestive of medullary sponge kidney. Medullary sponge kidney is characterized by cystic dilatation of collecting tubules. Urine stasis within the dilated tubules predisposes to infection and calculus formation within the dilated tubules or urinary tracts. On excretory urogram, contrast within the dilated tubules produces a striated ‘paintbrush’ appearance of the renal pyramids

111
Q
  1. A 60-year-old man has an unenhanced CT scan of renal tracts for suspected right renal colic. The examination is normal apart from an exophytic rounded lesion at the midpole of the left kidney, which is denser than adjacent renal parenchyma. You elect to perform an intravenous contrast enhanced examination in the nephrographic phase to further evaluate this lesion. Which of the following Hounsfield attenuation values would be most appropriate if this lesion is benign?

A. 20 pre-contrast attenuation, 30 post-contrast attenuation.

B. 30 pre-contrast attenuation, 60 post-contrast attenuation.

C. 40 pre-contrast attenuation, 50 post-contrast attenuation.

D. 50 pre-contrast attenuation, 80 post-contrast attenuation.

E. 60 pre-contrast attenuation, 70 post-contrast attenuation.

F. 70 pre-contrast attenuation, 100 post-contrast attenuation

A
  1. E. 60 pre-contrast attenuation, 70 post-contrast attenuation.

The attenuation of the normal renal parenchyma typically ranges from 30 to 40 HU. That of hyperattenuating renal masses usually is at least 40 HU but no higher than 90 HU on CT without intravenous contrast. Benign cysts are overwhelmingly the most common type of hyperattenuating renal mass and are also known as hyperdense renal cysts. They are usually cysts containing haemorrhage or proteinaceous material. Hyperattenuating cysts should not enhance and therefore cannot be diagnosed with confidence by using unenhanced CT alone. A proper CT examination includes scanning both before and after the administration of intravenous contrast material. Masses that increase in attenuation by 10 HU or less are considered non-enhancing. Masses that increase in attenuation by more than 10 HU are considered enhancing. However, because the standard deviation of attenuation measurements may be more than 10 HU, an attenuation difference of 20 HU or more is a more specific criterion of enhancement

112
Q
  1. A 24-year-old male patient presents to the A&E department with a history of severe episodic right-sided loin pain, radiating to the groin. He has no history of previous renal calculi. A low-dose non-contrast CT of the renal tracts shows a calculus in the distal right ureter, adjacent to the vesicoureteric junction. The calculus measures 7 mm in diameter. The patient has a horseshoe kidney. There is stranding in the peri-nephric fat and around the right ureter. The density of the calculus is measured to be 1500 HU. Which of these observations is least likely to have relevance to this patient’s treatment?

A. Site of the calculus.

B. Size of the calculus.

C. Perinephric and periureteric stranding.

D. Density of the calculus.

E. Horseshoe kidney.

A
  1. C. Perinephric and periureteric stranding.

Whilst this observation is sensitive and specific for detecting calculi, it does not have a bearing on stone outcome or urgency of treatment. The site is of value because if the stone impacts, it dictates the therapeutic approach. The size of the calculus can be used to assess the likelihood of spontaneous passage, with calculi over 5 mm more likely to cause obstruction. The density of the calculus has been found to predict the success of extracorporeal shock wave lithotripsy (ESWL). The presence of anatomical variations is also of interest to the urologists in treatment planning, if this becomes necessary.

113
Q
  1. A 54-year-old woman with no history of major illness is incidentally discovered to have a small, solid enhancing lesion on CT at the lower pole of the right kidney. The CT has been performed pre and post intravenous contrast. The lesion measures 9 mm in size. What is the most appropriate management for this lesion?

A. Nephron sparing surgery.

B. Percutaneous biopsy.

C. Repeat CT in 3–6 months.

D. Right nephrectomy.

E. Lesion ablation.

A
  1. C. Repeat CT in 3–6 months.

Solid masses smaller than 1 cm are challenging. Firstly, there is a reasonable chance that a very small solid mass is benign. Secondly, it is often difficult to characterize a mass smaller than 1 cm as solid and enhancing, despite a meticulous technique using state-of-the-art CT and MR imaging. Thirdly, these masses are often too small to biopsy, therefore when encountering a mass that is believed to be solid and is less than 1 cm in size, it is reasonable to observe them with an initial examination with CT or MR at 3–6 months followed by yearly examinations. A full work-up could ensue when the mass reaches 1 cm in size.

114
Q
  1. A 36-year-old labourer working on a building site falls 15 feet from scaffolding on his back onto a wheelbarrow. He is catheterized in the resuscitation room and is noted to have gross haematuria. Which of the following CT findings are consistent with a grade 4 renal injury?

A. Renal artery avulsion.

B. Shattered kidney.

C. Deep laceration to the collecting system.

D. Subcapsular haematoma.

E. Parenchymal contusion

A
  1. C. Deep laceration to the collecting system.

Renal injuries are classified into five grades of severity by the American Association for the Surgery of Trauma (AAST), with approximately 80% classified as grade 1 (Table 4.1). Most significant injuries manifest as haematuria and no significant urinary tract injury occurs in the absence of gross haematuria and shock. In addition, most significant renal trauma is associated with additional visceral injury.

115
Q
  1. An 18-year-old mountain bike enthusiast is suspected of sustaining a renal injury after attempting a front wheel touch-up manoeuvre. A laceration to the right kidney is noted on CT, which demonstrates contrast enhancement during the pyelographic phase of the examination. What is the significance of this finding?

A. Pre-existing angiomyolipoma.

B. Active haemorrhage.

C. Devascularization.

D. Renal infarction.

E. Urine leak.

A
  1. E. Urine leak.

Lacerations generally contain clotted blood and therefore do not enhance on scans obtained with intravenous contrast. Contrast enhancement during the pyelographic phase of the CTexamination indicates the presence of a urine leak. A delayed scan of 10–15 minutes may show the extent of the urinary extravasation. Intense enhancement within a laceration during the early phase indicates active haemorrhage. Focal areas of infarction do not enhance (unlike contusions). The cortical rim nephrogram is a sign of a devascularized kidney, which occurs due to laceration of the main renal artery.

116
Q
  1. A 45-year-old man has a complex cyst identified in the right kidney on an ultrasound scan performed to assess non-specific epigastric pain. He subsequently has a CT scan of kidneys carried out pre and post administration of intravenous contrast. You classify the complex cyst as IIF (II requiring follow-up) according to the Bosniak classification. Which of the following features is most likely seen at CT imaging?

A. Presence of calcification.

B. Thickened smooth wall with enhancement.

C. Multiple thin non-enhancing septa.

D. Hyperattenuating cyst <3 cm in size.

E. Thickened irregular wall.

A
  1. C. Multiple thin non-enhancing septa.

The criteria for a Bosniak IIF cyst are multiple hairline-thin septa with or without perceived (not measurable) enhancement, minimal smooth thickening of wall or septa that may show perceived (not measurable) enhancement, calcification may be thick and nodular but no measurable enhancement present, no enhancing soft-tissue components, and intrarenal non-enhancing high attenuation renal mass (> 3cm). The follow-up is CT or MRI at 6 months, 1 year, and then yearly up to 5 years. The criteria for a Bosniak II cyst that does not require follow-up include few hairline-thin septa with or without perceived (not measurable) enhancement, fine calcification or a short segment of slightly thickened calcification in the wall or septa, homogeneously high-attenuating masses (<3 cm) that are sharply marginated and do not enhance. Thickened irregular or smooth walls or septa, with measurable enhancement, are features ofa Bosniak III cyst and these are surgical lesions unless there are co-morbidities or limited life expectancy, when they may be observed

117
Q
  1. A 25-year-old male patient presents to A&E with left renal angle pain, radiating to the groin and significant haematuria on dipstick urinalysis. His kidneys, ureters, and bladder (KUB) x-ray is normal. He has a CT KUB, which shows no evidence of a calculus, but a mildly dilated left renal system as far as the mid ureter with peri-ureteric stranding to this point. Given the classical history, the patient undergoes an MR urogram, which shows a filling defect in the mid left ureter and a diagnosis of an obstructing ureteric calculus is made. The patient is observed and this diagnosis is confirmed when the patient passes a calculus the following day. The patient has a relevant past medical history, which led you to suspect a calculus in spite of the CT findings. Which of these clinical histories would lead you to suspect this?

A. Recurrent proteus infection.

B. HIV-positive patient on antiretroviral therapy.

C. Dehydrated patient.

D. Familial history of xanthine calculi.

E. Gout.

A
  1. B. HIV-positive patient on antiretroviral therapy.

Whilst CT KUB has 95–99% sensitivity for renal calculi, occasionally calculi due to antiretroviral therapy can be undetectable even to CT. Proteus is associated with formation of staghorn calculi and chronic dehydration with calcium oxalate calculi, both of which are radio-opaque. Xanthine calculi and urate calculi are classically not detectable on plain film KUB, but are usually visible on CT KUB.

118
Q
  1. A 46-year-old man presents with a 6-week history of painless haematuria.A CT urogram is performed and this demonstrates a 4-cm left renal mass, which is centrally placed involving both cortex and medulla. Which of the following findings will be most helpful when determining if this is a TCC invading the cortex, rather than a renal cell carcinoma invading the collecting system?

A. Presence of calcification.

B. Necrosis within the mass.

C. Renal vein invasion.

D. Maintenance of reniform shape.

E. Hypovascularity of lesion to renal parenchyma

A
  1. D. Maintenance of reniform shape.

Advanced TCC of the collecting system in the kidney can cause distortion of the renal architecture, but typically the reniform shape of the kidney is maintained. As RCC arises in the renal cortex, large tumours of this nature typically distort the normal renal outline. Calcification can rarely occur in both RCCs and TCCs, although perhaps more commonly in RCC. TCC demonstrates calcification in less than 3%, when it is usually diffuse and punctate. Necrosis can occur in both large RCCs and large TCCs, giving a heterogenous enhancement pattern on CT. Renal vein invasion is a more typical manifestation of RCC and both these tumours may be hypovascular compared to normal renal parenchyma on a nephrographic phase of the CT examination.

119
Q
  1. A 50-year-old man is discovered to have a cystic abnormality at the lower pole of the left kidney during an ultrasound scan performed for right upper quadrant pain. You think this might be a complex cystic lesion and therefore recommend a CT scan of renal tracts. This shows a conglomeration of variable sized cysts at the lower pole of the left kidney, but no capsule or mural irregularities of the cysts. What is the most likely diagnosis?

A. Localized cystic renal disease.

B. Multilocular cystic nephroma.

C. Renal lymphangiomatosis.

D. Cystic clear cell carcinoma.

E. Multicystic dysplastic kidney.

A
  1. A. Localized cystic renal disease.

This is an uncommon, non-familial, and non-progressive disorder of the kidney characterized by the replacement of all or localized areas of a kidney by multiple variably sized cysts. These cysts form clusters that are separated by thin areas of normal renal parenchyma. The aggregated cysts in localized cystic disease can frequently appear like a multi-septate mass, but they do not form a distinct encapsulated mass and do not show mural irregularities, which are characteristics of cystic neoplasms. Multilocular cystic nephroma in adults is much more common in females (female:male 9:1) and manifests as a multi-loculated cystic lesion with hair-like septa and minimal mural enhancement. It is distinguished from localized cystic disease by the presence of a capsule. Typically extension into the central renal sinus is found and multilocular cystic nephroma is often classified on imaging as Bosniak III lesions. Lymphangioma of the kidney is a rare benign cystic tumour that most often arises from the peripelvic region or renal sinus. It may more rarely arise from the lymphatics of the capsule or cortex. In the diffuse form of lymphangiomatosis, the cystic changes occur diffusely in the renal sinus or perinephric region, with a relatively normal appearing renal parenchyma. Clear cell RCC presents as a solid or a cystic lesion, the cystic variant accounting for 4–15% of all RCCs. These lesions typically have nodular or septal enhancement, distinguishing them from benign cystic renal lesions. Multicystic dysplastic kidney is a congenital maldevelopment in which the kidney is completely replaced by cysts with no normal renal parenchyma remaining

120
Q
  1. A 32-year-old patient with a history of renal failure has undergone a renal transplant in your centre. You are carrying out serial ultrasound investigations over the weekend to assess the success of the transplant. The graft was a cadaveric transplant. The initial ultrasound scan showed no evidence of perigraft collection, but did show mild renal pelvic dilatation. The pulsatility index (PI) was 2.0 and the resistive index was 0.9. The patient was oliguric. Due to the fears of acute rejection, the patient was commenced on cyclosporine and steroids. The patient, now day 4, is currently anuric and has developed pain over the graft site. You have done an ultrasound. The arterial flow in the kidney has changed. Proximally you notice reversal of diastolic flow evident on spectral waveforms, but in other areas there is no flow evident in the interlobar arteries. Flow is still present in the main renal artery. The RI is now 0.95 and the PI is unchanged. There is still mild renal pelvic dilatation, but you notice a tubular structure of intermediate echogenicity in the renal hilum. There is no evidence of perigraft collection. What do you think the most appropriate next investigative/ therapeutic step is?

A. Continue immunosuppression and further observation, with appropriate fluid administration.

B. Continue immunosuppression and renal biopsy.

C. Refer for immediate arterial intervention.

D. Refer for immediate venous intervention.

E. Nephrostomy.

A
  1. D. Refer for immediate venous intervention.

There are a number of complications to be aware of in the immediate post-transplant period. The most efficient and cost-effective method of surveillance for these is ultrasound. Acute rejection and acute tubular necrosis (ATN) both occur early in the post-transplant phase and can be difficult to differentiate. Both can cause elevation of pulsatility index (PI) (normal <1.5) and resistive index (RI) (normal <0.7), although these tend to be higher with rejection. ATN is more common with cadaveric transplants. It usually resolves within a matter of days or weeks, but ultimately biopsy can be necessary to reliably differentiate these processes. This patient probably had a degree of ATN initially. Arterial occlusion is uncommon and is evident by reduced/absent arterial flow in the main artery as well as in the interlobar arteries, which can demonstrate loss of flow with any cause of severe oedema (e.g. renal vein thrombosis or acute rejection). Renal vein thrombosis causes pain and anuria. It is associated with early treatment with cyclosporine and steroids. It also causes a rise in the RI and PI. The key feature to notice is the persistent flow in the main renal artery, reducing the likelihood of renal arterial thrombosis. There is also a tubular structure noted in the hilum, which is what a thrombosed renal vein looks like, and reversal of diastolic flow on arterial traces

121
Q
  1. A 38-year-old patient is referred for an urgent IVU and ultrasound by their GP, who has picked up mild renal impairment on recent blood tests. The ultrasound shows two normal-sized kidneys with no evidence of cortical loss. The IVU shows a normally enhancing renal outline, but the pelvicalyceal system is abnormal. Some calyces show a non-specific clubbed or blunted appearance. Thin tracks are noted extending from other calyces. In a number of calyces there is a filling defect noted within a rounded, blunted calyx. From these appearances you are able to make a diagnosis. The patient has a classical history for this condition. From the given options, what is the most typical history?

A. Recent history of acute hypotension on a background of dehydration.

B. Pyrexia, renal angle pain, and urine culture positive for E. coli.

C. Chronic non-steroidal anti-inflammatory drug (NSAID) analgesic overuse.

D. Previously diagnosed reflux nephropathy.

E. History of TCC within bladder.

A
  1. C. Chronic non-steroidal anti-inflammatory drug (NSAID) analgesic overuse.

The appearances described are those of renal papillary necrosis. This can be due to a number of causes, most commonly analgesic abuse, but also severe renal infection in diabetics, sickle cell disease, haemophilia, and renal vein thrombosis. The causes of chronic renal disease can be subdivided by the effects noted on the renal parenchyma and the papillary/pelvicalyceal system, and whether these are uni- or bilateral. In bilateral cases such as this, no radiological abnormality suggests possible glomerulonephritis, acute tubular necrosis (ATN), acute cortical necrosis, or pyelonephritis. Generalized infiltration of the renal parenchyma suggests amyloid or malignant infiltration (e.g. lymphoma or leukaemia). Papillary/pelvicalyceal abnormalities, in the presence of normal parenchyma, indicate papillary necrosis, medullary sponge kidney, or renal TB. Papillary calyceal abnormality with focal parenchymal loss indicates reflux nephropathy, TB, or calculus. Both parenchymal loss and papillary calyceal abnormalities indicate obstructive nephropathy or severe reflux nephropathy

122
Q
  1. You are asked to assess a renal transplant kidney in the early post-operative period for any abnormality and to serve as a baseline study. Which of the following findings on colour Doppler ultrasound with regard to the PI, RI and B mode ultrasound imaging, with regard to renal morphology, would be considered within normal limits?

A. PI < 1.5, RI < 0.7, loss of cortico-medullary differentiation.

B. PI < 1.5, RI < 0.7, prominence of renal pyramids.

C. PI > 1.8, RI < 0.7, loss of cortico-medullary differentiation.

D. PI > 1.8, RI < 0.7, prominence of renal pyramids.

E. PI < 1.5, RI > 0.9, loss of cortico-medullary differentiation.

F. PI < 1.5, RI > 0.9, prominence of renal pyramids.

G. PI > 1.8, RI > 0.9, loss of cortico-medullary differentiation.

H. PI > 1.8, RI > 0.9, prominence of renal pyramids

A
  1. B. PI < 1.5, RI < 0.7, prominence of renal pyramids.

PI < 1.5 or RI < 0.7 can be regarded as normal, whilst a PI > 1.8 or RI = 0.9 should be regarded as abnormal. Although both ATN and acute rejection cause the PI and RI to rise, the likelihood of acute rejection is greater with higher values. Complete absence of diastolic flow, or flow reversal, is due to acute rejection in the majority of cases. Morphologically, the transplant kidney is very similar to the native and many of the subtle differences are attributed to the improved resolution of the former. There is a well-defined renal parenchyma peripherally, with a highly reflective echogenic sinus centrally. In distinction to the native kidney, the renal pyramids are more commonly visualized within the transplant, being hypoechoic, relative to the parenchyma itself. Loss of cortico-medullary differentiation is pathological and is a B mode finding that can be associated with acute rejection.

123
Q

(MSK) 23 A 73-year-old man sustained a ureteric injury during a road traffic accident and is now haemodynamically unstable. What is the most appropriate radiological investigation in the acute setting?

a Contrast-enhanced CT

b Unenhanced CT

C Retrograde urethrography

d One-shot IVU following intravenous contrast medium administration

e Trans-rectal ultrasound (TRUS) of prostate to look for damage to prostatic urethra

A

23 Answer D: One-shot IVU following intravenous contrast medium administration

In an unstable patient, there is no time for a CT scan, and a `one-shot’ IVU would be the simplest and most effective study prior to potential surgery. CT remains the investigation of choice in any blunt abdominal trauma in a stable patient

124
Q

49 A 49-year-old man is scheduled for an intravenous urogram as part of the investigation of a known transitional cell carcinoma of the bladder. His control image is normal. Five minutes following the administration of 100 mL of non-ionic low osmolar iodinated contrast medium, at which time the patient is well, there is no opacification of either renal collecting system or ureter but faint nephrograms are present. What is the most likely cause for this appearance?

a Bilateral renal artery stenosis

b Bilateral synchronous renal transitional cell carcinomas causing obstruction

c Bilateral ureteric obstruction and hydronephrosis from the bladder primary

d Contrast extravasation

e Profound hypotension related to contrast injection

A

49 Answer C. Bilateral ureteric obstruction and hydronephrosis from the bladder primary

Collecting system opacification is delayed because of the raised backpressure reducing ultrafiltration of contrast at the glomerulus. Delayed imaging will be required to demonstrate the level of obstruction. Urgent urological opinion should be sought as ureteric stenting or nephrostomies may be required to relieve the obstruction

125
Q

51 Which of the following CT protocols is optimal for characterisation of a renal cortical solid mass seen on ultrasound in a 56-year-old man?

a Unenhanced scan, followed by post-intravenous contrast scans acquired at 60 and 100 seconds

b Unenhanced scan, followed by post-intravenous contrast scans acquired at 15 and 100 seconds

c Unenhanced scan, followed by post-intravenous contrast scans acquired at 90 seconds and 15 minutes

d Post-intravenous contrast scans acquired at 60 and 90 seconds

e Unenhanced scan, followed by post-intravenous contrast scans acquired at 90 seconds

A

51 Answer A: Unenhanced scan, followed by post-intravenous contrast scans acquired at 60 and 100 seconds

An unenhanced scan detects the presence of calcification, which may otherwise be obscured by contrast. The 60-second acquisition shows the corticomedullary phase, where there is differential enhancement of the cortex and the 100-second acquisition shows the nephrogram phase where the renal parenchyma uniformly enhances

126
Q

52 A 27-year-old male presents with severe left loin pain, colicky in nature and microscopic haematuria. Which of the following imaging techniques is the optimal method to prove or exclude the diagnosis of ureteric colic?

a Renal ultrasound

b Plain abdominal radiograph

c Unenhanced CT abdomen and pelvis, performed supine with normal mA dose protocol

d Unenhanced CT abdomen and pelvis, performed prone with low mA dose protocol

e Limited intravenous urogram

A

52 Answer D: Unenhanced CT abdomen and pelvis, performed prone with low mA dose protocol

Virtually all ureteric calculi, even those radiolucent on plain radiography, are detected by CT. A low mA dose protocol is performed without intravenous or oral contrast. The patient lies prone to distinguish between calculi at the vesicoureteric junction and those that have already passed into the bladder; the latter will lie dependently within the bladder.

127
Q

54 A 43-year-old man presents with right flank pain and haematuria. A CT KUB performed to exclude renal colic is negative for an obstructing ureteric calculus but instead shows a predominantly lowdensity solitary right renal mass. What feature on CT would be more in keeping with this lesion being a renal carcinoma than a sporadic angiomyolipoma?

a Presence of internal fat

b Presence of internal haemorrhage

C Size of 5 cm

d Presence of internal calcification

e Extension into peri-renal fat

A

54 Answer D: Presence of internal calcification

AML >4cm are symptomatic in >80%, and bleed spontaneously in 50-60%. Intratumoural calcifications are virtually never present with AML. Rarely renal cell carcinoma can undergo osseous metaplasia with growth of fatty marrow and associated calcifications.

128
Q

59 A 23-year-old female is demonstrated to have bilateral hyperdense renal masses on unenhanced CT. Following intravenous contrast administration the lesions remain homogeneous but are hypodense to normal renal cortex. There is associated retroperitoneal lymph node enlargement. What is the most likely diagnosis?

a Renal lymphoma

b Von Hippel-Lindau disease

C Adult polycystic kidney disease

d Angiomyolipomas

e Xanthogranulomatous pyelonephritis

A

59 Answer A: Renal lymphoma

Renal lymphoma is a common site for extra-nodal non-Hodgkin’s lymphoma. It usually shows poorer enhancement than renal cortex in the nephrographic phase.

129
Q

60 A 55-year-old male is 18 months following right radical nephrectomy for an 8-cm renal cell carcinoma, which invaded the renal vein (T3 b NO MO, Stage IIIA), and is undergoing a surveillance CT scan. Where is the most likely site for distant metastatic recurrence?

a Brain

b Contralateral kidney

C Lung

d Skin

e Bowel

A

60 Answer C: Lung

Recurrence of renal cell carcinoma usually occurs within the first six years after surgery. The risk increases with the initial stage. Distant metastases develop in 20-30% of patients, with lungs the commonest site (50-60%), followed by mediastinum, bone and liver.

130
Q

61 A 34-year-old male presents with left-sided renal colic and has a KUB that does not demonstrate any radio-opaque calculi. A CT is then performed and again no radio-opaque calculi are visible, but there is a small filling defect in the left ureter at the vesicoureteric junction on delayed postcontrast images. What is the most likely underlying diagnosis?

a Hyperparathyroidism

b Chronic urinary infection

C Gout

d Cystinuria

e Protease inhibitor use in HIV

A

61 Answer E: Protease inhibitor use in HIV

Indinavir (a protease inhibitor) use results in crystalline stones that are nonopaque on CT. They are demonstrated by contrast-enhanced CT as filling defects in the ureter.
* Hyperparathyroidism - calcium oxalate/phosphate calculi, HU>1000 * Chronic urinary infection - struvite (magnesium ammonium phosphate) calculi, HU300-900 * Gout - uric acid calculi, HU 150-500 (radiolucent on plain film) * Cystinuria - cysteine calculi, HU variable 200-880

131
Q

62 A 35-year-old female with renal impairment is demonstrated to have bilateral renal cortical tramline calcification on CT. What is the most likely diagnosis?

a Hyperparathyroidism

b Acute cortical necrosis

c Renal tubular acidosis

d Tuberculosis

e Renal artery aneurysm

A

62 Answer B: Acute cortical necrosis

Cortical nephrocalcinosis, described as tramline calcification, can be seen in any cause of acute cortical necrosis, including renal vein thrombosis, although obstetric shock is the commonest cause. Renal damage is more severe than in acute tubular necrosis.

132
Q

63 A 58-year-old male with no family history of renal cysts and no cysts in other organs is demonstrated to have multiple renal cysts on ultrasound. What is the most likely diagnosis?

a Acquired renal cystic disease

b Autosomal dominant polycystic kidney disease

C Multicystic dysplastic kidney

d Multiple simple cysts

e Von Hippel-Lindau disease

A

63 Answer D: Multiple simple cysts

Multiple simple cysts increase in frequency with age and are the most likely cause in a patient over 50 years with no family history of cysts or cysts in other organs

133
Q

64 A 27-year-old female with a family history of nephrectomy for malignancy is demonstrated to have multiple renal cysts on a surveillance ultrasound. What is the most likely diagnosis?

a Autosomal dominant polycystic kidney disease

b Multiple simple cysts

C Acquired renal cystic disease

d Cystic renal cell carcinoma

e Von Hippel-Lindau disease

A

64 Answer E: Von Hippel-Lindau disease

Von Hippel-Lindau disease is an autosomal-dominant disorder, manifestations of which include renal and pancreatic cysts, renal cell carcinoma, and cerebellar haemangioblastomas.

134
Q

65 A 35-year-old female undergoing investigation for hypertension and renal impairment is demonstrated with ultrasound to have multiple bilateral renal cysts as well as hepatic and pancreatic cysts. What is the most likely diagnosis?

a Conn’s syndrome

b Autosomal-dominant polycystic kidney disease

C Multicystic dysplastic kidney

d Multiple simple cysts

e Phaeochromocytoma

A

65 Answer B: Autosomal-dominant polycystic kidney disease

Autosomal-dominant polycystic kidney disease typically presents clinically with hypertension and renal failure aged 30-50 years, and cysts are present in the liver and pancreas, which along with a family history differentiates this from other causes.

135
Q

66 A 60-year-old male diabetic on haemodialysis for 10 years is demonstrated on ultrasound to have multiple small renal cysts, some of which have high echogenicity walls consistent with calcification. What is the most likely diagnosis?

a Acquired renal cystic disease

b Von Hippel-Lindau disease

C Multicystic renal cell carcinoma

d Multicystic dysplastic kidney

e Autosomal dominant polycystic kidney disease

A

66 Answer A: Acquired renal cystic disease

Acquired renal cystic disease occurs in 90% of patients who have been on dialysis for 5-10 years. Innumerable small cysts form and the kidney enlarges over time. Wall calcification is common and they regress following transplantation. Complications include haemorrhage and renal cell carcinoma

136
Q

68 A child is demonstrated to have increased echogenicity of the renal medulla on ultrasound, and plain radiography confirms the presence of nephrocalcinosis. What is the most likely cause?

a Renal tubular acidosis

b Hyperparathyroidism

C Medullary sponge kidney

d Renal papillary necrosis

e Primary hyperoxaluria

A

68 Answer A: Renal tubular acidosis

All are causes of medullary nephrocalcinosis, but distal renal tubular acidosis is the commonest cause in a child. It may be associated with rickets

137
Q

69 A 32-year-old male is a restrained passenger in a head-on motor vehicle collision and complains of left flank pain. He is assessed with a contrast-enhanced trauma CT. What feature on his CT would require urgent intervention (surgical/ radiological), assuming no other organ injury is identified?

a A contained leak of opacified urine into the retroperitoneum

b Peripheral renal enhancement

c A low-attenuation laceration extending to the medulla, but not to the collecting system

d A low-attenuation subcapsular haematoma >1 cm in depth

e A wedge-shaped area of low attenuation poorly enhancing renal cortex

A

69 Answer B: Peripheral renal enhancement

A stretching injury to the renal artery damages the intima and thrombus can then occlude the artery, which causes non-perfusion of the affected kidney. Capsular vessels do not arise from the main renal artery and so result in peripheral renal enhancement (the rim sign). The kidney may appear otherwise intact. Renal revascularisation should be accomplished within two hours of injury, but can be successful later depending on the collateral supply and extent of injury.

138
Q

70 A 23-year-old male motorcyclist is struck by a car at approximately 40 mph. He has bruising to the left flank and back and is tender in the left upper quadrant and lower chest wall. He is tachycardic but normotensive and has no haematuria. Which is the most appropriate imaging modality?

a FAST ultrasound

b Intravenous urogram

C Unenhanced CT KUB

d Abdominal and pelvic CT with intravenous contrast

e No imaging is required

A

70 Answer D: Abdominal and pelvic CT with intravenous contrast

Contrast-enhanced CT is the imaging modality of choice in a stable patient with or without haematuria and with evidence of major flank impact. Both US and IVU have a low sensitivity for detection of renal trauma, which often will not result in free intra-abdominal fluid. No imaging is required for an adult patient with microscopic haematuria, who is haemodynamically stable and with no other indication for CT. Instead observation until haematuria settles is appropriate. However, this does not hold true for the paediatric trauma patient, in whom microscopic haematuria without hypotension can be associated with significant renal imaging and CT imaging is warranted

139
Q

51 A 65-year-old woman becomes acutely short of breath one day following coronary angiography. A CTPA is performed and is negative for pulmonary embolism. Within two days her serum creatinine has risen. What would allow the diagnosis of contrast-induced nephrotoxicity to be made?

a Rise in serum creatinine from 80 pmol/L to 124 pmol/L

b Rise in serum creatinine from 100 1jmol/L to 120 imol/L

C Rise in serum creatinine from 100pmol/L to 144pmol/L and documented hypotension during coronary angiography

d Rise in serum creatinine from 80 pmol/L to 160 pmol/L with a large iatrogenic retroperitoneal haematoma

e Rise in serum creatinine from 200pmol/L to 244pmol/L with commencement of an angiotensin-converting enzyme inhibitor

A

51 Answer A: Rise in serum creatinine from 80pmol/L to 124pmol/L

The definition of contrast-induced nephropathy is an impairment of renal function (an increase in serum creatinine by more than 25 % or 44 pmol/L (0.5 mg/ dL)) which occurs within three days following intravascular administration of contrast medium, in the absence of an alternative aetiology

140
Q

52 A 74-year-old man is undergoing staging for renal cell carcinoma. In the assessment of which of the following might MRI be superior to CT?

a Local lymph node spread

b Parenchymal lung metastases

c Inferior vena caval invasion

d Characterisation of the primary tumour

e Differentiation between Ti and T2 disease

A

52 Answer C. Inferior vena caval invasion

The role of MRI in staging renal carcinoma is one of problem solving. Invasion of the renal vein and inferior vena cava is better shown on T1 W sequences with and without contrast, particularly in the coronal and sagittal planes.

141
Q

53 A 79-year-old woman is undergoing staging for a suspected renal primary malignancy. In the assessment of which of the following might 18FDG PET-CT have an additional role over other imaging modalities?

a Differentiating between transitional cell and renal cell carcinoma

b A 3-cm short axis regional lymph node

C Lytic bone metastases

d Inferior vena caval invasion

e Detection of synchronous primary tumours

A

53 Answer C: Lytic bone metastases

‘SFDG is not useful in primary renal or urothelial tumour assessment as it is physiologically excreted in the urine.’8FDG PET-CT is useful for the detection of and response to therapy of lytic bone metastases.

142
Q

58 An adult patient is demonstrated to have a normal-sized right kidney with an irregular apparently scarred cortex. Which one of the following sonographic features would indicate the presence of persistent foetal lobulation?

a Dromedary hump

b Broad cortical depression over a normal calyx

C Lateral indentation of the renal sinus

d Cortical depressions between calyces

e Multiple cortical depressions over dilated calyces

A

58 Answer D: Cortical depressions between calyces

Foetal lobulation occurs due to incomplete fusion of the foetal lobules, leading to a lobulated contour with depressions/interlobular septa occurring between calyces.

143
Q

59 A 51 year old has an ultrasound for non-specific abdominal pain and the renal cortex is noted to be hypoechoic relative to the liver parenchyma. What is the likely diagnosis?

a Cirrhosis

b Fatty liver

C Normal finding

d Diffuse metastatic disease

e Haemochromatosis

A

59 Answer C: Normal finding

The normal renal cortex is hypoechoic or isoechoic to the liver parenchyma. Arcuate arteries may be seen as small hyperechoic foci without shadowing at the corticomedullary junction. The medullary pyramids are hypoechoic to the renal cortex.

144
Q

60 Which of the following radiopharmaceuticals has high cortical binding and is therefore the agent of choice for evaluation of the renal cortex?

a I-131-orthoiodohippurate (Hippuran®)

b Cr-51-ethylenediaminetetraacetic acid (EDTA)

C Tc-99m-dimercaptosuccinic acid (DMSA)

d Tc-99m-diethylenetriamine pentaacetic acid (DTPA)

e Tc-99m-mercaptoacetyltriglycine (MAG3)

A

60 Answer C: Tc-99m-dimercaptosuccinic acid (DMSA)

Tc-99m-DMSA has minimal urinary excretion (<5%) and high cortical binding (50% of dose at four hours post injection), which makes it the ideal agent for assessment of renal cortical activity

145
Q

63 A 22-year-old female with a history of childhood urinary tract infections underwent an ultrasound for assessment of renal parenchymal disease. This demonstrated focal loss of renal parenchyma with underlying clubbed calyces in the right upper pole with normal appearances elsewhere. What is the most likely diagnosis?

a Glomerulonephritis

b Acute cortical necrosis

c Papillary necrosis

d Focal reflux nephropathy (chronic atrophic pyelonephritis)

e Focal infarction

A

63 Answer D: Focal reflux nephropathy (chronic atrophic pyelonephritis)

Focal reflux nephropathy is the only condition listed that gives a calyceal abnormality with associated cortical loss. It usually results as a consequence of infection associated with reflux in childhood. It may be widespread or focal in which case it is more common in the upper poles and on the right. The kidney is always small and there may be contralateral compensatory hypertrophy.

146
Q

64 A 26-year-old male with a history of recurrent haemoptysis and impaired renal function underwent an ultrasound for assessment of renal parenchymal disease. This demonstrated smooth small kidneys with no calyceal abnormality. What is the most likely diagnosis?

a Focal infarction

b Glomerulonephritis

c Amyloidosis

d Tuberculosis

e Papillary necrosis

A

64 Answer B: Glomerulonephritis

Goodpasture’s syndrome is the association of glomerulonephritis and pulmonary haemorrhage secondary to anti-glomerular basement membrane antibodies. The only renal imaging features are symmetrical change in size, initially enlarged, later small, with excessive renal sinus fat.

147
Q

65 A 56-year-old male on long-term antiretroviral medication presents with lower urinary tract symptoms, haematuria and sterile pyuria. Unenhanced CT shows unilateral focal high attenuation `cloudy’ dilated calyces with no associated cortical loss and delayed post-contrast imaging demonstrates poor opacification of the ipsilateral collecting system. What is the most likely diagnosis?

a Xanthogranulomatous pyelonephritis

b Renal stone disease

C Tuberculosis

d Acute papillary necrosis

e Candida albicans infection

A

65 Answer C: Tuberculosis

The clinical and radiological features suggest tuberculosis. Cloudy high attenuation dilated calyces represents calcification within caseous pyonephrosis. Calcification is seen in 30% of cases and may result in putty kidney or calcified autonephrectomy. One feature that suggests TB rather than papillary necrosis or focal reflux nephropathy is the relatively poor excretion of contrast in relation to the degree of obstruction.

148
Q

66 A 65-year-old male with chronic loin pain and recurrent urinary tract infections underwent an ultrasound which showed an enlarged left kidney with dilated calyces and low echogenicity parenchyma. A CT was then performed, which demonstrated multiple rounded fat attenuation masses replacing the renal parenchyma with associated perinephric fat stranding. Which of the followin additional findings strongly suggests xanthogranulomatous pyelonephritis as the cause?

a Avid cortical enhancement following contrast

b A heterogeneously enhancing focal renal mass

C Punctate renal calcification

d Hydronephrosis and hydroureter

e A renal pelvic calculus

A

66 Answer E: A renal pelvic calculus

Xanthogranulomatous pyelonephritis is an uncommon chronic inflammation usually associated with proteus infection and 70% have renal pelvic calculi, in which renal calyces and parenchyma are replaced by lipid-laden macrophages, debris and other inflammatory cells. The nephrogram is totally or focally absent in 80%. The generalised form is commoner but a focal form can mimic a renal tumour

149
Q

68 A 25-year-old female patient is under follow-up for post-operative surveillance of a live donor kidney transplant in the right iliac fossa and an ultrasound scan is being performed. What is the most likely anastomosis of the transplant renal artery?

a Aortic patch anastomosis to the external iliac artery

b End-to-side anastomosis to the external iliac artery

C End-to-end anastomosis to the external iliac artery

d End-to-side anastomosis to the common iliac artery

e End-to-end anastomosis to the internal iliac artery

A

68 Answer B: End-to-side anastomosis to the external iliac artery

The transplant kidney is usually placed in the contralateral iliac fossa extraperitoneal space. Usually, a live donor kidney is anastomosed end-to-side with the external iliac artery, or less commonly endto-end with the internal iliac artery. An aortic patch is usually removed with a cadaveric transplant; this is not possible with a living donor

150
Q

69 A 35-year-old female diabetic on dialysis undergoes a cadaveric renal transplant. Postoperatively there is primary non-function of the graft with no reduction in serum creatinine. Which of the following investigations reliably differentiates acute tubular rejection from acute rejection?

a Tc-99m-diethylenetriamine pentaacetic acid (DTPA) renal scintigraphy

b Duplex Doppler renal ultrasound

c MRI d Renal angiography

e Percutaneous renal biopsy

A

69 Answer E: Percutaneous renal biopsy

Biopsy is inevitably required to make the diagnosis of rejection. Acute tubular necrosis does have a characteristic pattern on dynamic renal scintigraphy, with almost normal first pass perfusion, blood pool uptake only at 80-180 seconds and no excretion. Rejection may be identified by reduced perfusion with or without swelling. Duplex US and MR findings are non-specific indicators of transplant dysfunction.

151
Q

70 A 42 year old with a live donor renal transplant undergoes Duplex Doppler US assessment for investigation of rising creatinine. The calculated resistance index (Pourcelot Index), RI is 0.9. How is this index calculated?

a Peak systolic velocity/End diastolic velocity

b (Peak systolic velocity - End diastolic velocity)/End diastolic velocity

c (Peak systolic velocity - End diastolic velocity)/Peak systolic velocity

d Peak systolic velocity/(Peak systolic velocity - End diastolic velocity)

e (Peak systolic velocity - End diastolic velocity)/Temporal mean velocity

A

70 Answer C: (Peak systolic velocity - End diastolic velocity)/Peak systolic velocity

The RI is sensitive to changes in downstream flow resistance and is used to assess changes in diastolic flow in low-resistance vascular beds. The normal limit for a renal transplant is 0.8. The pulsatility index (PI) is calculated by (Peak systolic velocity - End diastolic velocity)/Temporal mean velocity.

152
Q

(MSK) 22 A 65-year-old lady sustained an injury to her left loin by falling down a flight of stairs. She had a short hypotensive episode but responded well to IV fluids. She was noted to have bruising and severe pain in her loin extending to her back and urine dipstick showed presence of microscopic haematuria. What is the best investigation?

A Observation, no imaging required at present

b Intravenous urogram

c Contrast-enhanced CT

d Ultrasound

e Renal angiogram

A

22 Answer C: Contrast-enhanced CT

Multidetector CT imaging is the gold standard for diagnosing and assessing the extent of renal injury, peri-renal haemorrhage, extravasation of urine, pedicle injury, and associated solid-organ injury. Indications include blunt trauma with (macroscopic or microscopic) haematuria and hypotension, polytrauma, rapid deceleration and penetrating injury

153
Q

45 A 29-year-old female with a previous history of renal stone disease presents with acute left loin pain while 19 weeks’ pregnant. Ultrasound shows (maternal) left hydronephrosis. What is the best next line investigation?

a T2-weighted static fluid MR urography

b T1-weighted contrast-enhanced excretory MR urography

c Contrast-enhanced CT urography

d Unenhanced low-dose CT abdomen and pelvis

e Intravenous urography

A

45 Answer A: T2-weighted static fluid MR urography

MR urography is indicated as there is no risk of ionising radiation exposure to the foetus. Static fluid sequences are appropriate. Gadolinium contrast is contraindicated in pregnancy

154
Q

46 A 35-year-old woman with loin pain, dysuria and fever is diagnosed with acute uncomplicated ascending bacterial pyelonephritis. What appearance is likely to have been seen on CT?

a Cortical thinning

b Perinephric fat stranding

c Alternating bands of hypo- and hyperattenuation of the renal parenchyma

d Round peripheral hypoattenuating renal lesions

e Geographic low-attenuation lesion with peripheral enhancement

A

46 Answer C: Alternating bands of hypo- and hyperattenuation of the renal parenchyma

This appearance is analogous to the striated nephrogram seen on excretory urography and reflects the underlying pathology of tubular obstruction, interstitial oedema and vasospasm. On delayed (3-6 hours) imaging, the hypoattenuating regions show delayed and persistent enhancement due to prolonged accumulation and transit of contrast through the collecting system. In haematogenous seeding of pyelonephritis (from staphylococcal or streptococcal infections), round peripheral hypoattenuating lesions are seen. They can mimic neoplasia if pyelonephritis is not suspected clinically.

155
Q

48 A patient is being followed up after receiving a renal transplant. They are noted to have a dilated collecting system and rising creatinine. What is the most likely cause?

a Denervation of the renal collecting system

b Extrinsic compression from a perinephric fluid collection

c Ureteral calculi

d Ureteral ischaemia

e Ureteral kinking

A

48 Answer D: Ureteral ischaemia

Ninety per cent of post-transplant ureteral obstruction is due to ischaemia, usually at the distal ureter close to the ureterovesical junction. This is the area furthest from the renal artery. The other options are also less common causes of obstruction

156
Q

54 A 45-year old woman with tinnitus is referred for a contrast enhanced MRI for investigation of suspected acoustic neuroma. She is concerned regarding a `killer skin disease’ which she has read in the popular press is caused by MR contrast agents. Which of the following statements regarding nephrogenic systemic fibrosis (NSF) and the administration of gadolinium-based MR contrast media is correct? (as of February 2009)

a Gadolinium-based media have been proven to be responsible for causing NSF

b The incidence of NSF in at risk subjects given Gadodiamide (Omniscan®) is 25%

c Gadodiamide is contraindicated in patients with chronic kidney disease (stages 4 & 5) with a glomerular filtration rate (GFR) of <30mL/min

d Gadolinium-based media can be safely used as a replacement for iodinated contrast media to reduce contrast nephropathy

e Immediate dialysis following administration of gadolinium-based media prevents the development of NSF

A

54 Answer C: Gadodiamide is contraindicated in patients with a glomerular filtration rate (GFR) of <30mL/min

There is a causal link between gadolinium-based media and NSF, but no direct proof. There are >200 cases reported, all in patients with renal failure. No cases have been reported in patients with normal renal function. The incidence in at risk patients given Gadodiamide is 3-7 %. Gd-based agents are not recommended as replacements for iodinated contrast. The UK regulatory position as of February 2009 is summarised:
1 Gadodiamide and Gadopentetic acid (Magnevist®) are currently contraindicated in patients with GFR <30 mL/min, including those on dialysis. Gadodiamide is also contraindicated in all patients with renal impairment awaiting liver transplantation. Both agents should be used with caution in patients with GFR 30-59mL/min, & in children <1 year old. 2 Careful consideration should be given before using other Gd-based media in patients with GFR <30mL/min 3 All patients should be screened by history/laboratory tests before administration of Gd-based media 4 Haemodialysis after administration of Gd-based media may remove contrast but there is no evidence that this affects the development or treatment of NSF.

157
Q

55 A 35-year-old African male with AIDS and renal impairment has an ultrasound which shows bilateral enlarged echogenic kidneys with pelvicalyceal thickening. What is the most likely cause of his renal impairment?

a Hypertension unrelated to HIV status

b HIV-related nephropathy

c Highly active antiretroviral therapy (HAART) related renal side-effects

d Opportunistic infection with Pneurnocystis jirovecii (carinii)

e HIV-related malignancy (lymphoma/Kaposi sarcoma)

A

55 Answer B: HIV-related nephropathy

The US findings suggest HIV-related nephropathy, a form of glomerulosclerosis, which is the commonest cause of renal impairment in the HIV-positive population. It particularly affects young black males and is the third leading cause of end stage renal disease in African-Americans aged 20- 64 years. Malignancy, HAART therapy and P jirovecii associated nephrocalcinosis are all recognized renal manifestations of HIV infection.

158
Q

56 One week following a left laparoscopic partial nephrectomy a 55-year-old male has a CT for persistent pain at the surgical site. Which of the following findings would be most suspicious for a significant post-operative complication?

a Adherence of the kidney to the posterior abdominal wall, and surrounding reactive change

b Presence of a fat attenuation mass within the surgical bed

C Presence of a low-attenuation mass containing gas bubbles limited to the surgical bed

d Presence of a mass with attenuation of 50-60HU limited to the surgical bed

e A wedge-shaped non-enhancing parenchymal area

A

56 Answer E: A wedge-shaped non-enhancing parenchymal area

The renal arteries are clamped during the surgery. Damage to the intima may cause local thrombosis and wedge-shaped non-enhancing renal parenchymal infarcts. To promote haemostasis, fat or haemostatic material may be placed in the surgical bed. Fat may later be confused with an angiomyolipoma, and gas bubbles can be present with the latter which may be confused with a gas containing abscess in the early post-operative period. Residual haematoma may have an attenuation of 50-60HU.

159
Q

57 A 50-year-old man was found on CT scan to have an incidental renal mass. Biopsy subsequently showed this to be papillary renal cell carcinoma (RCC). What features are typical of this condition?

a It is one of the rarest forms of renal cell carcinoma

b It is typically less homogeneous and more avidly enhancing on CT than clear cell (conventional) RCC

c It is less frequently bilateral and multifocal than clear cell RCC

d It has a much better prognosis than clear cell RCC

e It arises from the epithelium of the medullary collecting duct

A

57 Answer D: It has a much better prognosis than conventional RCC

Papillary RCC is the second commonest form (10-15%) of RCC after clear cell RCC (70%). Both arise from the proximal convoluted tubule. It is more homogeneous and relatively hypovascular; this is a key imaging finding, and is more frequently multifocal and bilateral. It has the best prognosis of all forms of RCC.

160
Q

62 A 30-year-old female is being worked up for living donor laparoscopic nephrectomy. She asks why she needs further imaging after her contrast-enhanced CT scan. What further information is required that cannot be obtained from the CT?

a Number of renal arteries

b Length of the renal vein

C Split renal function

d Ureteral anatomy

e Presence of fibromuscular dysplasia

A

62 Answer C: Split renal function

Split renal function is assessed with Tc-99m-DMSA or Tc-99m-MAG3 scintigraphy. Dual-phase MDCT with 3-D post-processing is a quick and accurate method for living donor laparoscopic nephrectomy assessment. Of critical importance in the planning and performance of the operation is the renal arterial and venous anatomy

161
Q

64 You are asked to develop a protocol to minimise the incidence of contrastinduced nephropathy (CIN). What intervention has been proven to be of most benefit in its prevention?

a No treatments are proven to be effective

b N-acetylcysteine

c Monitoring of serum creatinine for 72 hours post procedure

d Periprocedural intravenous hydration

e High osmolar iodinated contrast media

A

64 Answer D: Periprocedural intravenous hydration

Randomised controlled studies and/or meta-analyses have demonstrated reduction in incidence of CIN with periprocedural hydration, low- versus high osmolar and iso- versus low osmolar contrast media. The role of N-acetylcysteine is currently inconclusive

162
Q

66 A43-year-old woman presents with a subarachnoid haemorrhage and a berry aneurysm is found, which is treated with endovascular coiling. She is incidentally discovered to have renal impairment and a renal ultrasound showed bilateral renal enlargement with multiple cortical cysts. Her father is known to have polycystic kidney disease. What is the likely method of inheritance of her condition?

a X-linked dominant

b X-linked recessive

C Autosomal dominant

d Autosomal recessive

e Mitochondrial

A

66 Answer C: Autosomal dominant

The patient has adult polycystic kidney disease which is autosomally dominantly inherited. Patients are also at risk of mitral valve prolapse and have an increased frequency of renal calculi, urinary tract infections and renal cell carcinoma. Bleeds into cysts are common and painful.

163
Q

68 A 17-year-old male was hit by a car and a contrast-enhanced trauma CT reveals a number of rightsided injuries. Images of his kidney show a lenticular-shaped area peripherally with a density of 50 HU. The remainder of the kidney enhances normally. What is the appropriate management of hisrenal I jury?

a Conservative management

b Endovascular embolisation/coiling

c Surgical revascularisation

d Endovascular stenting of renal artery

e Nephrectomy

A

68 Answer A: Conservative management

This is a typical description for a subcapsular haematoma which can usually be managed conservatively

164
Q

69 A 33-year-old female with systemic lupus erythematosus (SLE) presents with a painful mass in her left flank and haematuria. CT demonstrates non-opacification of the left renal vein. What is the likely appearance on ultrasound?

a Small kidney with enhanced cortico-medullary differentiation

b Small kidney with loss of cortico-medullary differentiation

c Normal-sized kidney with loss of cortico-medullary differentiation

d Enlarged kidney with enhanced cortico-medullary differentiation

e Enlarged kidney with loss of cortico-medullary differentiation

A

69 Answer E: Enlarged kidney with loss of cortico-medullary differentiation

The patient has a renal vein thrombosis and the kidney is enlarged due to oedema. There are often hyperechoic areas representing haemorrhage.

165
Q
  1. A 50-year-old man of Indian origin was involved in a road trafc accident. CT scan of the abdomen shows a small non-functional shrunken kidney with extensive dystrophic calcifcations. What is the most likely diagnosis?

(a) Renal tuberculosis

(b) Chronic renal failure

(c) Lymphoma

(d) Renal cell carcinoma

(e) Multicystic dysplastic kidney

A
  1. (a) Renal tuberculosis

This appearance is typical of a ‘putty kidney’. This results in a non-functional, shrunken kidney and autonephrectomy. Tuberculosis can involve any part of the genitourinary tract. Renal tuberculosis is usually seen secondary to haematogenous spread from lungs. Renal tuberculosis can manifest in a number of ways including as a renal mass.

166
Q
  1. A 40-year-old woman presents with nausea, vomiting and bilateral flank pain. Ultrasound of the kidneys, ureters and bladder shows loss of normal corticomedullary differentiation with lack of cortical blood flow. CT shows lack of cortical enhancement on both kidneys with enhancing medulla. Delayed images show no excretion of contrast into the collecting system. Te most likely diagnosis is?

(a) Medullary nephrocalcinosis

(b) Renal cortical necrosis

(c) Papillary necrosis

(d) Acute pyelonephritis

(e) Acute interstitial nephritis

A
  1. (b) Renal cortical necrosis

This is rare cause of acute renal failure with typical imaging features as given. CT is the most sensitive and specifc imaging for this condition. Absent opacifcation of the cortex with enhancement of juxtamedullary kidney and poor contrast excretion is diagnostic for this condition

167
Q
  1. An 18-year-old male presented with left ureteric colic. Intravenous urogram shows that both the kidneys are enlarged and there is elongation, displacement and deformity of the calices (spider leg appearance). What is the most likely diagnosis?

(a) Lymphoma

(b) Autosomal dominant polycystic kidney disease

(c) End-stage renal failure

(d) Renal vein thrombosis

(e) Autosomal recessive polycystic kidney disease

A
  1. (b) Autosomal dominant polycystic kidney disease

Intravenous urogram appearances are typical due to large cysts distorting the collecting system. Autosomal recessive polycystic kidney disease (infantile type) produces microscopic cysts and on intravenous urography, and shows an initial faint nephrogram with increasingly dense nephrogram.

168
Q
  1. A 60-year-old diabetic had a contrast–enhanced CT of the chest. He suffered mild skin eruptions and itching after the scan. Te next day, he presented to the Accident & Emergency Department with abdominal pain and anuria. Plain abdominal radiograph shows smooth large kidneys with dense bilateral nephrogram and absence of contrast in the collecting system. What is the most likely diagnosis?

(a) Acute tubular necrosis

(b) Acute glomerulonephritis

(c) Acute cortical necrosis

(d) Papillary necrosis

(e) Pyelonephritis

A
  1. (a) Acute tubular necrosis

This is secondary to temporary marked reduction in tubular blood flow. Contrast injection especially in diabetics with glomerulosclerosis is one of the known causes. Typically, the kidneys are enlarged due to interstitial oedema with an immediate dense and persistent nephrogram with absence of contrast from the collecting system

169
Q
  1. A 42-year-old man with a history of fts presents with recurrent abdominal pain. Ultrasound shows a 4 cm heterogenous lesion in the upper pole of the right kidney, with moderate vascularity. Contrast-enhanced CT shows that the lesion predominantly contains tissue with Hounsfeld units of -65 to -80. Other smaller such lesions were also seen in the left kidney. What is the most likely diagnosis?

(a) Multifocal renal cell carcinoma

(b) Angiomyolipomas

(c) Lymphoma

(d) Metastases

(e) Multiple lipomas

A
  1. (b) Angiomyolipomas

The lesions contain fat (negative Hounsfeld units) and show vascularity within. With the history of fts, tuberous sclerosis should be considered as a diagnosis

170
Q
  1. A 55-year-old woman presents to her general physician with left renal colic and chronic recurrent urinary tract infections. CT of the urinary tract shows a staghorn calculus in the left kidney. Post-contrast images demonstrate multiple low attenuation masses, almost replacing the renal parenchyma of left kidney with peripheral rim enhancement, and minimal contrast excretion from the left kidney on delayed phase images. What is the most likely diagnosis?

(a) Hydronephrotic kidney

(b) Renal cell carcinoma

(c) Xanthogranulomatous pyelonephritis

(d) Pyonephrosis

(e) Lobar nephronia

A
  1. (c) Xanthogranulomatous pyelonephritis

This is a chronic suppurative infection characterised by replacement of renal parenchyma by lipid-laden macrophages. In most cases there is also a staghorn calculus with renal enlargement and a rim of cortical enhancement (‘bear paw’ sign).

171
Q
  1. A 50-year-old man with an underlying condition presents with acute abdominal pain. CT shows haemorrhage involving the left kidney. What is the most unlikely diagnosis?

(a) Xanthogranulomatous pyelonephritis

(b) Renal cell carcinoma

(c) Multiple angiomyolipoma

(d) Polyarteritis nodosa

(e) Cortical cysts

A
  1. (a) Xanthogranulomatous pyelonephritis

Other options are all associated with bleeding or spontaneous haemorrhage.

172
Q
  1. Regarding medullary sponge kidney, which of the following are correct?

(a) The cystic areas in the medulla do not communicate with the collecting ducts.

(b) The changes are unilateral in 25% of cases.

(c) It is a feature of Meckel-Gruber syndrome.

(d) Medullary nephrocalcinosis is visible radiographically in 10% of cases.

(e) There is an association with Ehlers-Danlos syndrome.

A

Answers:

(a) Not correct

(b) Correct

(c) Not correct

(d) Not correct

(e) Correct

Explanation:

Medullary sponge kidney is a common sporadic condition affecting young to middle aged adults. There is dysplastic cystic dilatation of papillary and medullary collecting ducts. Meckel- Gruber syndrome is an autosomal recessive syndrome comprising of multicystic dysplastic kidneys, occipital encephalocele and polydactyly. Medullary nephrocalcinosis is visible radiographically in 40% - 80%.

173
Q
  1. Concerning renal papillary necrosis, which of the following are correct?

(a) Necrosis involves the overlying cortex in 10% of cases.

(b) It may result from aspirin use.

(c) A single papilla is affected in approximately 15% of cases.

(d) It is associated with a higher incidence of renal tract squamous cell carcinoma.

(e) Medullary sponge kidney is a cause.

A

Answers:

(a) Not correct

(b) Correct

(c) Correct

(d) Correct

(e) Not correct

Explanation:

Cortex is not involved in renal papillary necrosis. It involves pyramids and medullary papilla. Medullary sponge kidney causes renal papillary calculi not necrosis.

174
Q
  1. Which of the following are correct regarding renal cell carcinoma?

(a) About 75% of tumours less than 3cm in size are hyperechoic on ultrasonography.

(b) Von Hippel-Lindau syndrome is risk factor.

(c) Calcification and cystic change occur in the minority of cases.

(d) About 30% of tumours are hypovascular on angiography.

(e) The lungs are the most common site for metastatic spread.

A

Answers:

(a) Correct

(b) Correct

(c) Correct

(d) Not correct

(e) Correct

Explanation:

5% of RCC are hypovascular on angiography. Tumours cause neovascularity. Renal vein or IVC extension of tumour is also seen on angiograph

175
Q
  1. Concerning developmental abnormalities of the kidneys which of the following are correct:

(a) Accessory renal arteries normally enter the upper and mid poles.

(b) Duplication of the collecting system is seen in 10-12% of people.

(c) Horseshoe kidney is seen in 1 in 200 births.

(d) Horseshoe kidneys are more prone to trauma.

(e) Accessory renal arteries are less common in patients with horseshoe kidney.

A

Answers:

(a) Not correct

(b) Not correct

(c) Not correct

(d) Correct

(e) Not correct

Explanation:

Accessory renal arteries commonly enter the lower pole below the hilum and are more common in patients with horseshoe kidney. Incidence of horseshoe kidney is 1 in 700 births. Duplication of collecting system is seen in only 4% of people.

176
Q
  1. Which of the following are correct regarding urinary tract stones:

(a) Craniocaudal size is underestimated on CT.

(b) Urinary tract obstruction cannot be diagnosed on unenhanced CT.

(c) Pure matrix stones are not visualized on unenhanced CT.

(d) Renal failure is a common clinical presentation.

(e) A plain abdominal X-Ray (KUB) has a sensitivity of 80%.

A

Answers:

(a) Not correct

(b) Not correct

(c) Correct

(d) Not correct

(e) Not correct

Explanation:

Partial voluming tends to overestimate the size of stone on CT.Unenhanced CT shows ureteric and collecting system dilatation, nephromegaly and perinephric and periureteric fat standing, which are all signs of obstruction. lain X-ray KUB has only 45% - 58% sensitivity. Renal failure is rare and is secondary to bilateral obstructing calculi.

177
Q
  1. Which of the following are correct regarding contrast media nephrotoxicity (CMN):

(a) 0.45% saline infusion before and after contrast media injection reduces the risk of nephrotoxicity.

(b) Nephrotoxic effect of contrast media is dose dependent.

(c) Prophylactic haemodialysis prevents nephrotoxicity.

(d) Is usually benign and resolves within 1-2 weeks.

(e) Acetylcystein is a potent vasoconstrictor.

A

Answers:

(a) Correct

(b) Correct

(c) Not correct

(d) Correct

(e) Not correct

Explanation:

Hemodialysis can induce hypovolaemia which worsens renal ischemia. Acetylcystein is an antioxidant and scavenger of oxygen free radicals. It enhances the biological effect of the endogenous vasodilator nitric oxide.

178
Q
  1. Which of the following are correct regarding angiomyolipoma (AML):

(a) Is the most common benign tumour of the kidney in adults.

(b) A high attenuation lesion on unenhanced CT excludes the diagnosis.

(c) The absence of fat-content on unenhanced CT excludes the diagnosis.

(d) Prolonged enhancement on delayed post-contrast CT favours a diagnosis of AML rather than renal cell carcinoma.

(e) Calcification is a common feature.

A

Answers:

(a) Correct

(b) Not correct

(c) Not correct

(d) Correct

(e) Not correct

Explanation:

AML with minimal fat content can have high attenuation on unenhanced CT. Fat content is characteristic of AML but 5% have predominance of blood vessels or immature fat. Calcification is a rare feature of AML.

179
Q
  1. Which of the following are correct regarding genitourinary tuberculosis (TB):

(a) Is the most common manifestation of extrapulmonary TB.

(b) In renal TB: calyceal dilatation is usually accompanied by pelvic dilatation.

(c) The endometrium is affected in >70% of women with genital TB.

(d) Papillary necrosis is a recognized feature.

(e) A raised CA-125 level excludes tubo-ovarian TB.

A

Answers:

(a) Correct

(b) Not correct

(c) Correct

(d) Correct

(e) Not correct

Explanation:

In renal TB earliest IVU abnormality is a ‘moth eaten’ calyx due to erosion, followed by papillary necrosis. Dilatation of one or more calyces without pelvic dilatation typically occurs due to infundibular stenosis. Tubo-ovarian involvement is usually caused by hematogenous or lymphatic spread, mimicking ovarian cancer clinically and radiologically. Diagnosis is generally made post-operatively.

180
Q
  1. Which of the following are correct regarding autosomal dominant polycystic kidney disease (ADPKD):

(a) 35-50% of patients develop end-stage renal failure.

(b) 2.5% of nephrons are affected by cystic change.

(c) Hepatic cysts are seen in 5-10% by 60 years of age.

(d) 2-10% of patients sustain subarachnoid haemorrhage.

(e) Cyst density >30 HU on post-intravenous contrast CT suggests malignant change.

A

Answers:

(a) Correct

(b) Correct

(c) Not correct

(d) Correct

(e) Not correct

Explanation:

Hepatic cysts are seen in 70%-75% by 60 yrs of age. The most common cause of increased density of cyst is haemorrhage (HU of 50-90). ADKPD patients on dialysis have an increased risk of cancer relative to population.

181
Q
  1. Which of the following are correct regarding causes of cortical nephrocalcinosis:

(a) Chronic glomerulonephritis.

(b) Hyperparathyroidism.

(c) Chronic transplant rejection.

(d) Acute cortical necrosis.

(e) Hypervitaminosis D.

A

Answers:

(a) Correct

(b) Not correct

(c) Correct

(d) Correct

(e) Not correct

Explanation:

Medullary nephrocalcinosis is seen in hyperparathyroidism, hypervitaminosis D, renal tubular acidosis, medullary sponge kidney, renal papillary necrosis, hypercalciuria and primary hyperoxaluria